Med Surg 2 - Chap 45, 6infection dew, IGGY CH. 23 CARE OF PTS WITH INFECTION, Ch 37. Care of Patients with Shock, Chapter 67 Sepsis, Ch 66 Shock, Sepsis, & MODS, Med Surg: Chapter 66 Critical Care, PEDS: Chapter 54: The Child with a Developmental Dis...

अब Quizwiz के साथ अपने होमवर्क और परीक्षाओं को एस करें!

A nurse gives a nasogastric feeding to a preterm male infant. As the mother watches, she asks, "Would it hurt my baby to suck on a pacifier during the feeding?" How should the nurse best respond? "It's difficult to determine the color of his lips while he's sucking on a pacifier. We'd rather wait until he's a little older." "If you want, he can suck on a pacifier now, but he may have problems later when he starts to suck from the breast or bottle." "Sucking on a pacifier during tube feedings may help him associate sucking with food so that he'll adjust better to oral feedings." "There's no real benefit in using a pacifier. Also, there's a relationship between using a pacifier and the development of buck teeth."

"Sucking on a pacifier during tube feedings may help him associate sucking with food so that he'll adjust better to oral feedings." The pacifier may satisfy nonnutritive sucking needs and stimulate flow of saliva and digestive juices. Protruding ("buck") teeth are associated with thumb sucking. Sucking on a pacifier promotes adaptation later to the breast or bottle; it does not hamper it. There is no evidence that a preterm infant's care is jeopardized by nonnutritive sucking.

Which of the following clinical manifestations does the nurse associate with rubeola? 1 Macular rash 2 Paroxysmal cough 3 Enlarged parotid glands 4 Generalized vesicular lesions

1 Rubeola (measles) starts with a discrete maculopapular rash on the face that spreads downward, eventually becoming confluent. A paroxysmal cough occurs with whooping cough. Enlarged parotid glands occur with mumps. Generalized vesicular lesions occur with chickenpox.

Which are examples of actively acquired specific immunity? Select all that apply. 1 Recovery from measles 2 Recovery from chickenpox 3 Maternal immunoglobulin in the neonate 4 Immunization with live or killed vaccines 5 Injection of human gamma immunoglobulin

1, 2, 4 Naturally acquired active-type immunity is seen in a client who has recovered from measles or chickenpox or who has been immunized with a live- or killed-virus vaccine. Maternal immunoglobulin in a neonate and an injection of human gamma immunoglobulin into a client are examples of passively acquired specific immunity.

A primary healthcare provider diagnoses late-stage (tertiary) syphilis in a client. Which statement made by the client supports this diagnosis? 1 "I noticed a wart on my penis." 2 "I have sores all over my mouth." 3 "I've been having a sore throat lately." 4 "I'm having trouble keeping my balance."

4 Neurotoxicity, as manifested by ataxia (balance problems), is evidence of tertiary syphilis, which may involve the central nervous system (CNS) or cardiovascular system. A wart on the penis occurs in the secondary stage of syphilis. Sores all over the mouth occur in the first and secondary stage of syphilis. Sore throat with flulike symptoms occurs in the secondary stage of syphilis.

While attempting to revive a person brought to the emergency department with extreme hypothermia (rectal temperature of 94° F), unconscious, and shivering, the nurse is alarmed when assessment reveals that the patient has: a. reddened, cold ears. b. stopped shivering. c. blanched, hardened fingers. d. blisters on hands and arms.

ANS: B Cessation of shivering indicates that the body's homeostatic response to generate heat has ceased and the patient's condition is deteriorating.

A client with active genital herpes has a cesarean birth. The nurse teaches the mother how to limit transmission of the virus to her newborn. The nurse concludes that the instructions have been understood when the mother makes what statement? 1 "I should avoid kissing the baby on the lips." 2 "I have to wear gloves when I'm holding the baby." 3 "I should wash my clothes and my baby's clothes separately." 4 "I have to wash my hands with soap and water before handling the baby."

4 The herpes virus disintegrates rapidly on contact with soap used in meticulous hand washing. The lesion is in the genital area, not on the lips; kissing will not affect the infant. Wearing gloves when holding the infant is not necessary; nor is washing the infant's clothes separately.

The nurse explains that a vaccination provides defense against infection via which type of immunity? a.Innate immunity b.The inflammatory response c.Antibody-mediated immunity d.Cell-mediated immunity

ANS: C Vaccinations produce an antibody-mediated immunity by stimulating the host to develop specific antibodies against specific diseases. PTS: 1 DIF: Cognitive Level: Knowledge REF: 105, Table 6-3 OBJ: 2 (theory) TOP: Immune Response KEY: Nursing Process Step: Implementation MSC: NCLEX: Health Promotion and Maintenance

How should the home health nurse advise the patient to treat a fever of 100° F? a.Take aspirin as needed. b.Take Tylenol every 4 to 6 hours. c.Bathe in cool water before bed. d.Do nothing at all.

ANS: D Allowing reasonable levels of fever allows the body's natural defenses to make a hostile environment to the pathogen through heat. PTS: 1 DIF: Cognitive Level: Application REF: 103 OBJ: 3 (theory) TOP: Fever KEY: Nursing Process Step: Implementation MSC: NCLEX: Physiological Integrity: Physiological Adaptation

A student nurse asks the nursing instructor why older adults are more prone to infection than other adults. What reasons does the nursing instructor give? (Select all that apply.) a. Age-related decrease in immune function b. Decreased cough and gag reflexes c. Diminished acidity of gastric secretions d. Increased lymphocytes and antibodies e. Thinning skin that is less protective

A, B, C, E Older adults have several age-related changes making them more susceptible to infection, including decreased immune function, decreased cough and gag reflex, decreased acidity of gastric secretions, thinning skin, and fewer lymphocytes and antibodies.

When all five of the contestants in a custard pie-eating contest arrive at the emergency department with vomiting and diarrhea, the nurse suspects that these signs are related to the contamination of the pies by _______________.

ANS: Staphylococcus aureus Staphylococcus aureus is the prime offender in contamination of custards, mayonnaise, and processed foods.

2. A nurse is caring for a patient with shock of unknown etiology whose hemodynamic monitoring indicates BP 92/54, pulse 64, and an elevated pulmonary artery wedge pressure. Which collaborative intervention ordered by the health care provider should the nurse question? a. Infuse normal saline at 250 mL/hr. b. Keep head of bed elevated to 30 degrees. c. Hold nitroprusside (Nipride) if systolic BP <90 mm Hg. d. Titrate dobutamine (Dobutrex) to keep systolic BP >90 mm Hg.

ANS: A The patient's elevated pulmonary artery wedge pressure indicates volume excess. A saline infusion at 250 mL/hr will exacerbate the volume excess. The other actions are appropriate for the patient. DIF: Cognitive Level: Apply (application) REF: 1633 TOP: Nursing Process: Planning MSC: NCLEX: Physiological Integrity

The nurse is caring for several patients and determines which patient to be most at risk for developing an infection related to a decreased anti-inflammatory response? a.A patient who has been experiencing high levels of stress for the last 3 months. b.A patient with a glycosylated Hgb level of 6.7%. c.A patient with osteoarthritis who was recently diagnosed. d.A patient who is scheduled for laparoscopic cholecystectomy in 2 weeks related to gallstones.

ANS: A The presence of increased levels of cortisol resulting from ongoing stress inhibits the anti-inflammatory response, thus making this patient most susceptible to developing an infection. A glycosylated Hgb level of 6.7% is normal; osteoarthritis and gallstones would not significantly increase a patient's likelihood of developing an infection. PTS: 1 DIF: Cognitive Level: Application REF: 100, Box 6-1, 103 OBJ: 10 (clinical) TOP: Cortisol KEY: Nursing Process Step: Implementation MSC: NCLEX: Health Promotion and Maintenance

3. The nurse gets the hand-off report on four clients. Which client should the nurse assess first? a. Client with a blood pressure change of 128/74 to 110/88 mm Hg b. Client with oxygen saturation unchanged at 94% c. Client with a pulse change of 100 to 88 beats/min d. Client with urine output of 40 mL/hr for the last 2 hours

ANS: A This client has a falling systolic blood pressure, rising diastolic blood pressure, and narrowing pulse pressure, all of which may be indications of the progressive stage of shock. The nurse should assess this client first. The client with the unchanged oxygen saturation is stable at this point. Although the client with a change in pulse has a slower rate, it is not an indicator of shock since the pulse is still within the normal range; it may indicate the client's pain or anxiety has been relieved, or he or she is sleeping or relaxing. A urine output of 40 mL/hr is only slightly above the normal range, which is 30 mL/hr.

The nurse recognizes indications of respiratory distress, which include: (Select all that apply.) a. gasping. b. wheezing. c. stridor. d. choking. e. stupor.

ANS: A, B, C, D All options except stupor are indicators of respiratory distress.

1. The student nurse studying shock understands that the common manifestations of this condition are directly related to which problems? (Select all that apply.) a. Anaerobic metabolism b. Hyperglycemia c. Hypotension d. Impaired renal perfusion e. Increased perfusion

ANS: A, C The common manifestations of shock, no matter the cause, are directly related to the effects of anaerobic metabolism and hypotension. Hyperglycemia, impaired renal function, and increased perfusion are not manifestations of shock.

2. The nurse caring for hospitalized clients includes which actions on their care plans to reduce the possibility of the clients developing shock? (Select all that apply.) a. Assessing and identifying clients at risk b. Monitoring the daily white blood cell count c. Performing proper hand hygiene d. Removing invasive lines as soon as possible e. Using aseptic technique during procedures

ANS: A, C, D, E Assessing and identifying clients at risk for shock is probably the most critical action the nurse can take to prevent shock from occurring. Proper hand hygiene, using aseptic technique, and removing IV lines and catheters are also important actions to prevent shock. Monitoring laboratory values does not prevent shock but can indicate a change.

Which areas of the body is/are protected by normal flora? (Select all that apply.) a.Skin b.Bladder c.Lower gastrointestinal (GI) tract d.Nose and throat e.Eye

ANS: A, C, D, E Normal flora inhabit and protect the skin, lower GI tract, nose and throat, and eyes. The bladder does not have any natural flora for protection. PTS: 1 DIF: Cognitive Level: Comprehension REF: 100, Table 6-1 OBJ: 2 (theory) TOP: Natural Flora KEY: Nursing Process Step: Implementation MSC: NCLEX: Physiological Integrity: Physiological Adaptation

Anticipatory guidance for the family of a preadolescent with a cognitive dysfunction should include information about a. Institutional placement b. Sexual development c. Sterilization d. Clothing

ANS: B Feedback A Preadolescence does not require the child to be institutionalized. B Preadolescents who have a cognitive dysfunction may have normal sexual development without the emotional and cognitive abilities to deal with it. It is important to assist the family and child through this developmental stage. C Sterilization is not an appropriate intervention when a child has a cognitive dysfunction. D By the time a child reaches preadolescence, the family should have received counseling on age-appropriate clothing.

Appropriate interventions to facilitate socialization of the cognitively impaired child include a. Providing age-appropriate toys and play activities b. Providing peer experiences, such as scouting, when older c. Avoiding exposure to strangers who may not understand cognitive development d. Emphasizing mastery of physical skills because they are delayed more often than verbal skills

ANS: B Feedback A Providing age-appropriate toys and play activities is important. However, peer interactions will better facilitate social development. B The acquisition of social skills is a complex task. Children of all ages need peer relationships. Parents should enroll the child in preschool. When older, they should have peer experiences similar to other children such as group outings, Boy and Girl Scouts, and Special Olympics. C Parents should expose the child to strangers so that the child can practice social skills. D Verbal skills are delayed more than physical skills.

The infant with Down syndrome is closely monitored during the first year of life for what serious condition? a. Thyroid complications b. Orthopedic malformations c. Dental malformation d. Cardiac abnormalities

ANS: D Feedback A Infants with Down syndrome are not known to have thyroid complications. B Orthopedic malformations may be present, but special attention is given to assessment for cardiac and gastrointestinal abnormalities. C Dental malformations are not a major concern compared with the life-threatening complications of cardiac defects. D The high incidence of cardiac defects in children with Down syndrome makes assessment for signs and symptoms of these defects important during the first year. Clinicians recommend the child be monitored frequently throughout the first 12 months of life, including a full cardiac workup.

The father of a child recently diagnosed with developmental delay is very rude and hostile toward the nurses. This father was cooperative during the child's evaluation a month ago. What is the best explanation for this change in parental behavior? a. The father is exhibiting symptoms of a psychiatric illness. b. The father may be abusing the child. c. The father is resentful of the time he is missing from work for this appointment. d. The father is experiencing a symptom of grief.

ANS: D Feedback A One cannot determine that a parent is exhibiting symptoms of a psychiatric illness on the basis of a single situation. B The scenario does not give any information to suggest child abuse. C Although the father may have difficulty balancing his work schedule with medical appointments for his child, a more likely explanation for his behavior change is that he is grieving the loss of a normal child. D After a child is diagnosed with a developmental delay, families typically experience a cycle of grieving that is repeated when developmental milestones are not met.

Many of the physical characteristics of Down syndrome present feeding problems. Care of the infant should include a. Delaying feeding solid foods until the tongue thrust has stopped b. Modifying diet as necessary to minimize the diarrhea that often occurs c. Providing calories appropriate to child's age d. Using special bottles that may assist the infant with feeding

ANS: D Feedback A The child has a protruding tongue, which makes feeding difficult. The parents must persist with feeding while the child continues the physiologic response of the tongue thrust. B The child is predisposed to constipation. C Calories should be appropriate to the child's weight and growth needs, not age. D Breastfeeding may not be possible if the infant's muscle tone or sucking reflex is immature. Mothers should be encouraged to pump breast milk and use special bottles for assistance with feeding. Some children with Down syndrome can breastfeed adequately.

Which action by the nurse is most helpful to prevent clients from acquiring infections while hospitalized? a. Assessing skin and mucous membranes b. Consistently using appropriate hand hygiene c. Monitoring daily white blood cell counts d. Teaching visitors not to visit if they are ill

B Consistent practice of proper hand hygiene is the best method to prevent infection, as most healthcare-associated infections are due to staff members' contaminated hands. Assessing the client and monitoring laboratory values will help the nurse catch signs of infection quickly but will not prevent infection from occurring. Teaching visitors not to come see the client when they are ill will also help prevent infection, but not to the degree that hand hygiene will.

A nurse is caring for preterm infants with respiratory distress in the neonatal intensive care unit. What is the priority nursing action? A.) Limiting caloric intake to decrease metabolic rate B.) Maintaining the prone position to prevent aspiration C.) Limiting oxygen concentration to prevent eye damage D.) Maintaining a high-humidity environment to promote gas exchange

D -- The moisture provided by the humidity liquefies the tenacious secretions, making gas exchange possible. Caloric intake is increased; the amount, number, and type of feedings are related to the metabolic rate. Infants should be placed in a side-lying rather than a prone position; the prone position is associated with apnea and sudden infant death syndrome. Limiting oxygen concentration to prevent eye damage is not a routine action; the concentration of oxygen depends on the oxygen concentration of the neonate's blood gases.

A male born at 28 weeks' gestation weighs 2 lb 12 oz. What does the nurse expect to note when performing an assessment? a.) Staring eyes B.) Absence of lanugo C.) Descended testicles D.) Transparent red skin

D --Transparent red skin is expected because of the absence of subcutaneous fat tissue. Preterm infants born nearer to term have open, staring eyes. Preterm infants generally are born with large amounts of lanugo, which begins to thin just before term and by 40 weeks is found only on the shoulders, back, and upper arms. The preterm infant's scrotum is small and the testicles usually are high in the inguinal canal

The nurse is caring for a client in the first stage of labor. Which position is the least desirable for the client if she is experiencing lower back pain? Sitting Supine Knee-chest Left side-lying

Supine Low back pain is aggravated when the client is in the supine position because of increased pressure from the fetus as the head rotates. A sitting position relieves back pain. The knee-chest position is an alternate position that a client may choose to use when laboring. The left side-lying position relieves back pain.

A 35-year-old client is scheduled for a vaginal hysterectomy. She asks the nurse about the changes she should expect after surgery. How should the nurse respond? "You will stop ovulating." "Surgical menopause will happen immediately." "Sexual intercourse will be uncomfortable when you resume it." "A hysterectomy doesn't affect the chronological age when menopause usually occurs."

"A hysterectomy doesn't affect the chronological age when menopause usually occurs." As the term hysterectomy implies, only the uterus is removed. The ovaries remain; therefore the client will experience menopause around the same time as women who have functioning ovaries. The client will ovulate and will not experience surgical menopause, because the ovaries are not removed with a hysterectomy. There should be no discomfort if there is an appropriate period of healing before the resumption of sexual intercourse.

A client with tuberculosis is prescribed rifampin. What does the nurse teach the client about this medication? Select all that apply. 1 "Avoid drinking alcohol while you are on this drug." 2 "Report immediately if you find a yellow appearance to the skin." 3 "Wear a protective clothing and sunscreen when going out in sunlight." 4 "Your soft contact lenses will become permanently stained with this drug." 5 "Immediately consult your physician if you find reddish orange tinge in your urine."

1, 2, 4 Rifampin is an antitubercular drug that kills slow-growing organisms residing in the caseating granulomas. Rifampin may cause liver damage, so alcohol should be avoided as it potentiates liver damage. Yellow appearance to the skin is a sign of liver failure. Therefore, a client on rifampin therapy is taught to report the presence of any yellowing of the skin. Rifampin permanently stains soft contact lenses and therefore the client is made aware to avoid wearing them while on the medication. Pyrazinamide causes photosensitivity reactions and therefore a client on that drug therapy is advised to wear protective clothing and sunscreen when going outdoors. The nurse should inform the client that rifampin changes the color of body secretions, which is normal and harmless.

A nurse is caring for a client with an infection caused by group A beta-hemolytic streptococci. The nurse should assess this client for responses associated with which illness? 1 Hepatitis A 2 Rheumatic fever 3 Spinal meningitis 4 Rheumatoid arthritis

2 Rheumatic fever Antibodies produced against group A beta-hemolytic streptococci sometimes interact with antigens in the heart's valves, causing damage and symptoms of rheumatic heart disease; early recognition and treatment of streptococcal infections have limited the occurrence of rheumatic heart disease. Hepatitis A, an inflammation of the liver, is caused by the hepatitis A virus (HAV), not by bacteria. The most common causes of meningitis, an infection of the membranes surrounding the brain and spinal cord, include Streptococcus pneumoniae, Neisseria meningitidis, and Haemophilus influenzae. Rheumatoid arthritis is believed to be an autoimmune disorder; it is not caused by microorganisms.

The nurse differentiates the types of treatment that are appropriate for each type of shock. Match the type of shock with the type of treatment associated with it. (Use each option one time only.) a. Cardiogenic shock b. Hypovolemic shock c. Anaphylactic shock d. Neurogenic shock e. Insulin shock 29. Administration of epinephrine 30. Administration of vasoconstrictors 31. Administration of glucose orally or IV 32. Administration of fluids 33. Administration of inodilators

29. C 30. D 31. E 32. B 33. A

The nurse is caring for four clients in the medical unit. Which nursing instruction indicates a need for correction? 1 Client A Abnormal vaginal bleeding. Avoid super absorbent tampons 2 Client B Diarrhea. Wash hands frequently 3 Client C AIDS. Never share your eating utensils. 4 Client D Tuberculosis. Wear a mask during transport to other areas.

3 Client C Human immunodeficiency virus leads to acquired immunodeficiency syndrome. The virus in client C cannot be transmitted through sharing eating utensils, hugging, dry kissing, shaking hands, and using toilet seats. The nurse should advise client A to use sanitary pads rather than superabsorbent tampons to prevent toxic shock syndrome due to Staphylococcus aureus infection. Client B with diarrhea should wash hands frequently to reduce the transmission of the disease. The nurse should advise client D with tuberculosis to wear a mask to prevent the transmission of Mycobacterium tuberculosis from small droplets when being transported.

When assessing the oral cavity of a newly admitted client with acquired immunodeficiency syndrome (AIDS), the nurse identifies areas of white plaque on the client's tongue and palate. What is the nurse's initial response? 1 Instruct the client to perform meticulous oral hygiene at least once daily. 2 Scrape an area of one of the lesions and send the specimen for a biopsy. 3 Document the presence of the lesions, describing their size, location, and color. 4 Consider that these lesions are universally found in clients with AIDS and require no treatment.

3 Documentation of nursing findings during assessment is a nursing function; this facilitates early treatment. Scraping an area of one of the lesions and sending the specimen for a biopsy medical intervention is beyond the scope of nursing practice. Inadequate oral hygiene has not been identified as a cause of plaques; once-daily treatment is insufficient for anyone. Candida is a frequent secondary infection in clients with AIDS; it is treated when present.

An adolescent comes to the school nurse complaining of a 2-day history of low-grade fever, exhaustion, and lack of energy and appetite. He has been tardy to school twice in the past week. Which assessment should the nurse use to identify the possible origin of the problem? 1 Eliciting the Kernig sign 2 Eliciting the Brudzinski sign 3 Checking for lymphadenopathy 4 Checking the pupillary response to light and accommodation

3 Checking for lymphadenopathy Infectious mononucleosis is caused by the Epstein-Barr virus. Mononucleosis is common in people between the age of 15 and 30 years. Signs and symptoms of mononucleosis include fever, fatigue, swollen lymph glands, and enlargement of the liver and spleen. Pupillary response to light and accommodation is checked as part of a neurologic assessment. The Kernig sign (asking the child to straighten a leg that is bent at a 90-degree angle at the knee) and Brudzinski sign (asking a child who is lying flat to bend his head and try to put his chin on his chest) are elicited as part of the assessment when meningitis is suspected.

A client reports neck stiffness, severe headache, and a decreased level of consciousness. What condition does the nurse suspect? 1 Encephalitis 2 Brain abscess 3 Viral meningitis 4 Bacterial meningitis

4 Bacterial meningitis is caused by a bacterium such as Streptococcus pneumonia. Fever, severe headache, neck stiffness, photophobia, and decreased levels of consciousness are symptoms that indicate bacterial meningitis. Encephalitis is the acute inflammation of brain. Nausea and vomiting are symptoms of encephalitis. Headache, fever, nausea, and vomiting are the symptoms of brain abscess. Headache, fever, and photophobia are the symptoms of viral meningitis.

A male client with small overgrowths on the skin in the cervical region arrives at the hospital. The laboratory report reveals the presence of human papillomavirus type 16. Which condition is associated with this virus? 1 Balanitis 2 Prosthitis 3 Genital warts 4 Penile carcinoma

4 Human papillomavirus type 16 is commonly associated with penile carcinoma. Uncircumcised males may be at higher risk for infections such as balanitis and prosthitis. Human papillomavirus types 11 and 6 are commonly associated with genital warts.

A nurse suspects that a newborn has toxoplasmosis, one of the TORCH infections. How and when may it have been transmitted to the newborn? A.) In utero through the placenta B.) In the postpartum period through breast milk C.) During birth through contact with the maternal vagina D.) After the birth through a blood transfusion given to the mother

A --Toxoplasmosis is caused by a parasitic protozoon that is acquired from inadequately cooked contaminated food or through handling of infected cat feces; the most common form of transmission to the newborn is by way of placental perfusion when in utero. There is no evidence that toxoplasmosis is transmitted in breast milk. The newborn does not contract toxoplasmosis from the maternal genital tract during the birth process. There is no evidence that toxoplasmosis is transmitted in blood transfused into the mother.

A preterm infant, born at 30 weeks' gestation, is receiving an intravenous electrolyte solution at a rate of 20 mL/hr by way of an umbilical arterial line. At the hourly intake measurement, the nurse observes that 40 mL has infused in the past hour. What is the nurse's first intervention? A.) Taking the vital signs B.) Comparing the intake with the ouput C.) Checking the practitioner's prescriptions D.) Slowing the infusion rate to half of the prescribed rate

A ---The priority is assessing the infant for circulatory overload; changes in the vital signs may indicate a problem that must be addressed quickly. Comparing the intake and output record wastes valuable time that should be spent assessing the infant's response. Checking the practitioner's prescription wastes valuable time that should be spent assessing the infant's response. After the infant's response is assessed, an adjustment of the IV rate may be prescribed.

The nurse is assessing a 12-hour-old newborn. What clinical finding should be reported to the health care provider? A.) Jaundice B.)Cephalhematoma C.) Erythema toxicum D.) edematous genitalia.

A --Jaundice occurring in the first 24 hours of life is pathological; it is associated with Rh or another blood incompatibility. Cephalhematoma is a collection of blood between the skull and periosteum that does not cross the suture line; it resolves within 6 weeks, and although it should be documented it does not require treatment. Erythema toxicum is newborn dermatitis, believed to be an inflammatory response. The rash is harmless, and although it should be documented it does not require treatment. Edematous genitalia, a response to maternal hormones, are common in newborns.

A nursing manager is concerned about the number of infections on the hospital unit. What action by the manager would best help prevent these infections? a. Auditing staff members' hand hygiene practices b. Ensuring clients are placed in appropriate isolation c. Establishing a policy to remove urinary catheters quickly d. Teaching staff members about infection control methods

A All methods will help prevent infection; however, health care workers' lack of hand hygiene is the biggest cause of healthcare-associated infections. The manager can start with a hand hygiene audit to see if this is a contributing cause.

A student nurse asks why brushing clients' teeth with a toothbrush in the intensive care unit is important to infection control. What response by the registered nurse is best? a. "It mechanically removes biofilm on teeth." b. "It's easier to clean all surfaces with a brush." c. "Oral care is important to all our clients." d. "Toothbrushes last longer than oral swabs."

A Biofilms are a complex group of bacteria that function within a slimy gel on surfaces such as teeth. Mechanical disruption (i.e., toothbrushing with friction) is the best way to control them. The other answers are not accurate.

A hospitalized client is placed on Contact Precautions. The client needs to have a computed tomography (CT) scan. What action by the nurse is most appropriate? a. Ensure that the radiology department is aware of the isolation precautions. b. Plan to travel with the client to ensure appropriate precautions are used. c. No special precautions are needed when this client leaves the unit. d. Notify the physician that the client cannot leave the room for the CT scan.

A Clients in isolation should leave their rooms only when necessary, such as for a CT scan that cannot be done portably in the room. The nurse should ensure that the receiving department is aware of the isolation precautions needed to care for the client. The other options are not needed.

A client has a wound infection to the right arm. What comfort measure can the nurse delegate to the unlicensed assistive personnel (UAP)? a. Elevate the arm above the level of the heart. b. Order a fan to help cool the client if feverish. c. Place cool, wet cloths on top of the wound. d. Take the client's temperature every 4 hours.

A Elevating the extremity above the level of the heart will help with swelling and pain. Fans are not recommended as they can disperse microbes. Having a cool, wet cloth on the wound may macerate the broken skin. Taking the client's temperature provides data but does not increase comfort.

A client is hospitalized and on multiple antibiotics. The client develops frequent diarrhea. What action by the nurse is most important? a. Consult with the provider about obtaining stool cultures. b. Delegate frequent perianal care to unlicensed assistive personnel. c. Place the client on NPO status until the diarrhea resolves. d. Request a prescription for an anti-diarrheal medication.

A Hospitalized clients who have three or more stools a day for 2 or more days are suspected of having infection with Clostridium difficile. The nurse should inform the practitioner and request stool cultures. Frequent perianal care is important and can be delegated but is not the priority. The client does not necessarily need to be NPO; if the client is NPO, the nurse ensures he or she is getting appropriate IV fluids to prevent dehydration. Anti-diarrheal medication may or may not be appropriate, and the diarrhea serves as the portal of exit for the infection.

A client has been admitted to the hospital for a virulent infection and is started on antibiotics. The client has laboratory work pending to determine if the diagnosis is meningitis. After starting the antibiotics, what action by the nurse is best? a. Assess the client frequently for worsening of his or her condition. b. Delegate comfort measures to unlicensed assistive personnel. c. Ensure the client is placed on Contact Precautions. d. Restrict visitors to the immediate family only.

A Meningitis is a disease caused by endotoxins, which are released with cell lysis. Antibiotics often work by lysing cell membranes, which would increase the amount of endotoxin present in the client's body. The nurse should carefully monitor this client for a worsening of his or her condition. Delegating comfort measures is appropriate for any client. Clients with meningitis are placed on Droplet Precautions, and initiating isolation should have been done on admission. The client does not need to have visitors restricted.

A nurse is caring for a client who has methicillin-resistant Staphylococcus aureus (MRSA) infection cultured from the urine. What action by the nurse is most appropriate? a. Prepare to administer vancomycin (Vancocin). b. Strictly limit visitors to immediate family only. c. Wash hands only after taking off gloves after care. d. Wear a respirator when handling urine output.

A Vancomycin is one of a few drugs approved to treat MRSA. The others include linezolid (Zyvox) and ceftaroline fosamil (Teflaro). Visitation does not need to be limited to immediate family only. Hand hygiene is performed before and after wearing gloves. A respirator is not needed, but if splashing is anticipated, a face shield can be used.

The student nurse learns that effective antimicrobial therapy requires which factors to be present? (Select all that apply.) a. Appropriate drug b. Proper route of administration c. Standardized peak levels d. Sufficient dose e. Sufficient length of treatment

A, B, D, E In order to be effective, antimicrobial therapy must use the appropriate drug in a sufficient dose, for a sufficient length of time, and given via the appropriate route. Some antimicrobials do require monitoring for peak and trough levels, but not all.

A client is being admitted with suspected tuberculosis (TB). What actions by the nurse are best? (Select all that apply.) a. Admit the client to a negative-airflow room. b. Maintain a distance of 3 feet from the client at all times. c. Order specialized masks/respirators for caregiving. d. Other than wearing gloves, no special actions are needed. e. Wash hands with chlorhexidine after providing care.

A, C A client with suspected TB is admitted to Airborne Precautions, which includes a negative-airflow room and special N95 or PAPR masks to be worn when providing care. A 3-foot distance is required for Droplet Precautions. Chlorhexidine is used for clients with a high risk of infection.

1. A client with severe sepsis has a serum lactate level of 6.2 mmol/L. The client weighs 250 pounds. To infuse the amount of fluid this client requires in 24 hours, at what rate does the nurse set the IV pump? (Record your answer using a whole number.) ____ mL/hr

ANS: 142 mL/hr The client weighs 250 pounds = 113.63636 kg. The fluid requirement for this client is 30 mL/kg = 3409 mL. To infuse this amount over 24 hours, set the pump at 142 mL/hr (3409/24 = 142).

SHORT ANSWER 1. A 198-lb patient is to receive a dobutamine infusion at 5 mcg/kg/minute. The label on the infusion bag states: dobutamine 250 mg in 250 mL normal saline. When setting the infusion pump, the nurse will set the infusion rate at how many mL per hour?

ANS: 27 In order to administer the dobutamine at the prescribed rate of 5 mcg/kg/minute from a concentration of 250 mg in 250 mL, the nurse will need to infuse 27 mL/hour. DIF: Cognitive Level: Apply (application) REF: 1642 TOP: Nursing Process: Implementation MSC: NCLEX: Physiological Integrity

In which order will the nurse perform the following actions when caring for a patient with possible C5 spinal cord trauma who is admitted to the emergency department? (Put a comma and a space between each answer choice [A, B, C, D, E].) a. Infuse normal saline at 150 mL/hr. b. Monitor cardiac rhythm and blood pressure. c. Administer O2 using a nonrebreather mask. d. Immobilize the patient's head, neck, and spine. e. Transfer the patient to radiology for spinal computed tomography (CT).

ANS: D, C, B, A, E The first action should be to prevent further injury by stabilizing the patient's spinal cord if the patient does not have penetrating trauma. Maintenance of oxygenation by administration of 100% O2 is the second priority. Because neurogenic shock is a possible complication, monitoring of heart rhythm and BP are indicated followed by infusing normal saline for volume replacement. A CT scan to determine the extent and level of injury is needed once initial assessment and stabilization are accomplished.

OTHER 1. The health care provider orders the following interventions for a 67-kg patient who has septic shock with a BP of 70/42 mm Hg and oxygen saturation of 90% on room air. In which order will the nurse implement the actions?(Put a comma and a space between each answer choice [A, B, C, D, E].) a. Obtain blood and urine cultures. b. Give vancomycin (Vancocin) 1 g IV. c. Start norepinephrine (Levophed) 0.5 mcg/min. d. Infuse normal saline 2000 mL over 30 minutes. e. Titrate oxygen administration to keep O2 saturation >95%.

ANS: E, D, C, A, B The initial action for this hypotensive and hypoxemic patient should be to improve the oxygen saturation, followed by infusion of IV fluids and vasopressors to improve perfusion. Cultures should be obtained before administration of antibiotics. DIF: Cognitive Level: Analyze (analysis) REF: 1645 OBJ: Special Questions: Prioritization TOP: Nursing Process: Implementation MSC: NCLEX: Physiological Integrity

The health care provider orders the following interventions for a 67-kg patient who has septic shock with a blood pressure of 70/42 mm Hg and O2 saturation of 90% on room air. In which order will the nurse implement the actions? (Put a comma and a space between each answer choice [A, B, C, D, E].) a. Give vancomycin 1 g IV. b. Obtain blood and urine cultures c. Start norepinephrine 0.5 mcg/min. d. Infuse normal saline 2000 mL over 30 minutes. e. Titrate oxygen administration to keep O2 saturation above 95%.

ANS: E, D, C, B, A The initial action for this hypotensive and hypoxemic patient should be to improve the O2 saturation, followed by infusion of IV fluids and vasopressors to improve perfusion. Cultures should be obtained before giving antibiotics.

The bacteria that are rod-shaped are classified as _________.

ANS: bacilli Bacilli are rod-shaped bacteria. PTS: 1 DIF: Cognitive Level: Knowledge REF: 101 OBJ: 1 (theory) TOP: Bacteria KEY: Nursing Process Step: N/A MSC: NCLEX: Health Promotion and Maintenance

The nurse explains that an infection occurring in the body represents an interrelationship between the __________, __________, and __________.

ANS: host, agent, environment host, environment, agent agent, host, environment agent, environment, host environment, host, agent environment, agent, host A pathologic agent, upon entering the body, must attach to a host in order to multiply in a supportive environment. PTS: 1 DIF: Cognitive Level: Application REF: 100, Box 6-1 OBJ: 1 (theory) TOP: Infection Process KEY: Nursing Process Step: Implementation MSC: NCLEX: Health Promotion and Maintenance

The nurse explains that the four lines of defense the body employs to combat infection are inflammatory response, immune response, __________, and __________.

ANS: skin, normal flora normal flora, skin The body is defended against infection by the skin, normal flora, and inflammatory and immune responses. PTS: 1 DIF: Cognitive Level: Comprehension REF: 102 OBJ: 2 (theory) TOP: Defense Against Infection KEY: Nursing Process Step: Implementation MSC: NCLEX: Physiological Integrity: Physiological Adaptation

Unlike fragile X syndrome, which affects primarily males, __________ (RS) is almost exclusively linked to female gender.

ANS: Rett syndrome An estimated 1:10,000 to 1:15,000 females are affected. RS is characterized by an initial period of normal development with symptoms emerging between the ages of 6 and 18 months. Social and intellectual development stops and seizures along with physical disabilities emerge.

A patient's vital signs are pulse 87, respirations 24, and BP of 128/64 mm Hg and cardiac output is 4.7 L/min. The patient's stroke volume is _____ mL. (Round to the nearest whole number.)

ANS: 54 Stroke volume = cardiac output/heart rate

The nurse is caring for a patient who has an intraortic balloon pump (IABP) following a massive heart attack. When assessing the patient, the nurse notices blood backing up into the IABP catheter. In which order should the nurse take the following actions? (Put a comma and a space between each answer choice [A, B, C, D].) a. Ensure that the IABP console has turned off. b. Assess the patient's vital signs and orientation. c. Obtain supplies for insertion of a new IABP catheter. d. Notify the health care provider of the IABP malfunction.

ANS: A, B, D, C Blood in the IABP catheter indicates a possible tear in the balloon. The console will shut off automatically to prevent complications such as air embolism. Next, the nurse will assess the patient and communicate with the health care provider about the patient's assessment and the IABP problem. Finally, supplies for insertion of a new IABP catheter may be needed, based on the patient assessment and the decision of the health care provider.

When assisting with oral intubation of a patient who is having respiratory distress, in which order will the nurse take these actions? (Put a comma and a space between each answer choice [A, B, C, D, E].) a. Obtain a portable chest-x-ray. b. Position the patient in the supine position. c. Inflate the cuff of the endotracheal tube after insertion. d. Attach an end-tidal CO2 detector to the endotracheal tube. e. Oxygenate the patient with a bag-valve-mask device for several minutes.

ANS: E, B, C, D, A The patient is pre-oxygenated with a bag-valve-mask system for 3 to 5 minutes before intubation and then placed in a supine position. Following the intubation, the cuff on the endotracheal tube is inflated to occlude and protect the airway. Tube placement is assessed first with an end-tidal CO2 sensor, then with a chest x-ray.

The nurse is planning care for a patient and determines that Expanded Precautions are warranted when performing care for a patient with which infection? a.Active tuberculosis (TB) b.Bacterial pneumonia c.A urinary tract infection (UTI) caused by E. coli d.A fungal infection of the groin and axilla

ANS: A Active TB can be spread by airborne pathogens. Masks and gowns, in addition to gloves, should be worn while caring for such patients. Standard Precautions would be used for patients with bacterial pneumonia, a UTI caused by E. coli, and a fungal infection of the groin and axilla. PTS: 1 DIF: Cognitive Level: Application REF: 109, Table 6-5 OBJ: 5 (theory) TOP: Expanded Precautions KEY: Nursing Process Step: Planning MSC: NCLEX: Safe, Effective Care Environment: Safety and Infection Control

11. Norepinephrine (Levophed) has been prescribed for a patient who was admitted with dehydration and hypotension. Which patient data indicate that the nurse should consult with the health care provider before starting the norepinephrine? a. The patient's central venous pressure is 3 mm Hg. b. The patient is in sinus tachycardia at 120 beats/min. c. The patient is receiving low dose dopamine (Intropin). d. The patient has had no urine output since being admitted.

ANS: A Adequate fluid administration is essential before administration of vasopressors to patients with hypovolemic shock. The patient's low central venous pressure indicates a need for more volume replacement. The other patient data are not contraindications to norepinephrine administration. DIF: Cognitive Level: Apply (application) REF: 1644 TOP: Nursing Process: Implementation MSC: NCLEX: Physiological Integrity

The nurse discusses and demonstrates proper hand hygiene to an immunocompromised patient and his wife. Which statement indicates a need for additional teaching? a."It is okay for my wife to wear artificial nails as long as she washes her hands properly." b."I should always wash my hands before I eat." c."Hand gels work as well as handwashing under most circumstances." d."I should use friction and wash my hands for about 20 seconds if I am using soap and water."

ANS: A Artificial nails harbor microorganisms regardless of good hand hygiene. Washing hands prior to eating is good practice, as well as using friction and washing for 15 to 30 seconds with soap and water. Hand gels are effective in most circumstances except for certain infections such as C. difficile and C. albicans. PTS: 1 DIF: Cognitive Level: Application REF: 107-108 OBJ: 2 (clinical) TOP: Hand Hygiene KEY: Nursing Process Step: Evaluation MSC: NCLEX: Safe, Effective Care Environment: Safety and Infection Control

A patient with septic shock has a BP of 70/46 mm Hg, pulse of 136 beats/min, respirations of 32 breaths/min, temperature of 104°F, and blood glucose of 246 mg/dL. Which intervention ordered by the health care provider should the nurse implement first? a. Give normal saline IV at 500 mL/hr. b. Give acetaminophen (Tylenol) 650 mg rectally. c. Start insulin drip to maintain blood glucose at 110 to 150 mg/dL. d. Start norepinephrine to keep systolic blood pressure above 90 mm Hg.

ANS: A Because of the decreased preload associated with septic shock, fluid resuscitation is the initial therapy. The other actions also are appropriate, and should be initiated quickly as well.

15. A patient with septic shock has a BP of 70/46 mm Hg, pulse 136, respirations 32, temperature 104° F, and blood glucose 246 mg/dL. Which intervention ordered by the health care provider should the nurse implement first? a. Give normal saline IV at 500 mL/hr. b. Give acetaminophen (Tylenol) 650 mg rectally. c. Start insulin drip to maintain blood glucose at 110 to 150 mg/dL. d. Start norepinephrine (Levophed) to keep systolic blood pressure >90 mm Hg.

ANS: A Because of the low systemic vascular resistance (SVR) associated with septic shock, fluid resuscitation is the initial therapy. The other actions also are appropriate, and should be initiated quickly as well. DIF: Cognitive Level: Apply (application) REF: 1644-1645 OBJ: Special Questions: Prioritization TOP: Nursing Process: Implementation MSC: NCLEX: Physiological Integrity

The nurse is caring for a patient who has septic shock. Which assessment finding is most important for the nurse to report to the health care provider? a. Skin cool and clammy c. Blood pressure of 92/56 mm Hg b. Heart rate of 118 beats/min d. O2 saturation of 93% on room air

ANS: A Because patients in the early stage of septic shock have warm and dry skin, the patient's cool and clammy skin indicates that shock is progressing. The other information will also be reported, but does not indicate deterioration of the patient's status.

The following interventions are ordered by the health care provider for a patient who has respiratory distress and syncope after eating strawberries. Which will the nurse complete first? a. Give epinephrine. b. Administer diphenhydramine. c. Start continuous ECG monitoring. d. Draw blood for complete blood count (CBC)

ANS: A Epinephrine rapidly causes peripheral vasoconstriction, dilates the bronchi, and blocks the effects of histamine and reverses the vasodilation, bronchoconstriction, and histamine release that cause the symptoms of anaphylaxis. The other interventions are also appropriate but would not be the first ones completed.

While assessing an obese resident in a long-term care facility, the nurse finds a red, moist rash under the patient's breasts, in the axilla, and in the inguinal fold. Based on this assessment, the nurse reports to the charge nurse that the resident probably has which type of infection? a.A fungal infection b.A bacterial infection c.An allergic reaction d.Contact dermatitis

ANS: A Fungal infections thrive in warm, moist environments and most frequently affect the skin. PTS: 1 DIF: Cognitive Level: Comprehension REF: 102 OBJ: 1 (theory) TOP: Fungi KEY: Nursing Process Step: Assessment MSC: NCLEX: Safe, Effective Care Environment: Safety and Infection Control

A 78-kg patient with septic shock has a pulse rate of 120 beats/min with low central venous pressure and pulmonary artery wedge pressure. Urine output has been 30 mL/hr for the past 3 hours. Which order by the health care provider should the nurse question? a. Administer furosemide (Lasix) 40 mg IV. b. Increase normal saline infusion to 250 mL/hr. c. Give hydrocortisone (Solu-Cortef) 100 mg IV. d. Titrate norepinephrine to keep systolic blood pressure (BP) above 90 mm Hg.

ANS: A Furosemide will lower the filling pressures and renal perfusion further for the patient with septic shock. Patients in septic shock require large amounts of fluid replacement. If the patient remains hypotensive after initial volume resuscitation with minimally 30 mL/kg, vasopressors such as norepinephrine may be added. IV corticosteroids may be considered for patients in septic shock who cannot maintain an adequate BP with vasopressor therapy despite fluid resuscitation.

Chapter 67: Nursing Management: Shock, Systemic Inflammatory Response Syndrome, and Multiple Organ Dysfunction Syndrome Chapter 67: Nursing Management: Shock, Systemic Inflammatory Response Syndrome, and Multiple Organ Dysfunction Syndrome Test Bank MULTIPLE CHOICE 1. A 78-kg patient with septic shock has a urine output of 30 mL/hr for the past 3 hours. The pulse rate is 120/minute and the central venous pressure and pulmonary artery wedge pressure are low. Which order by the health care provider will the nurse question? a. Give PRN furosemide (Lasix) 40 mg IV. b. Increase normal saline infusion to 250 mL/hr. c. Administer hydrocortisone (Solu-Cortef) 100 mg IV. d. Titrate norepinephrine (Levophed) to keep systolic BP >90 mm Hg.

ANS: A Furosemide will lower the filling pressures and renal perfusion further for the patient with septic shock. The other orders are appropriate. DIF: Cognitive Level: Apply (application) REF: 1640 TOP: Nursing Process: Implementation MSC: NCLEX: Physiological Integrity

5. A client is in shock and the nurse prepares to administer insulin for a blood glucose reading of 208 mg/dL. The spouse asks why the client needs insulin as the client is not a diabetic. What response by the nurse is best? a. "High glucose is common in shock and needs to be treated." b. "Some of the medications we are giving are to raise blood sugar." c. "The IV solution has lots of glucose, which raises blood sugar." d. "The stress of this illness has made your spouse a diabetic."

ANS: A High glucose readings are common in shock, and best outcomes are the result of treating them and maintaining glucose readings in the normal range. Medications and IV solutions may raise blood glucose levels, but this is not the most accurate answer. The stress of the illness has not "made" the client diabetic.

The nurse is caring for a patient with general sepsis. Which finding should first alert the nurse to a potential complication that warrants immediate attention? a.Increased lethargy b.Sudden coughing c.Elevated blood pressure d.Cloudy urine

ANS: A Increasing lethargy is an indicator of impending septic shock. Coughing and cloudy urine are not signs of impending septic shock. Decreased rather than increased blood pressure would indicate impending septic shock. PTS: 1 DIF: Cognitive Level: Application REF: 115 OBJ: 1 (theory) TOP: Septic Shock KEY: Nursing Process Step: Assessment MSC: NCLEX: Health Promotion and Maintenance

MULTIPLE CHOICE The nurse is teaching a patient about infection prevention. The nurse points out that covering the mouth and nose with a tissue for a sneeze reduces the probability of infection spreading by which route? a.Droplet b.Airborne c.Direct contact d.Indirect contact

ANS: A Infection from the droplet route requires the pathogens be expelled in droplets from the host and inhaled by another host. PTS: 1 DIF: Cognitive Level: Comprehension REF: 101 OBJ: 1 (theory) TOP: Disease-Producing Pathogens KEY: Nursing Process Step: Implementation MSC: NCLEX: Safe, Effective Care Environment: Safety and Infection Control

A patient who is receiving positive pressure ventilation is scheduled for a spontaneous breathing trial (SBT). Which finding by the nurse is most important to discuss with the health care provider before starting the SBT? a. New ST segment elevation is noted on the cardiac monitor. b. Enteral feedings are being given through an orogastric tube. c. Scattered rhonchi are heard when auscultating breath sounds. d. HYDROmorphone (Dilaudid) is being used to treat postoperative pain.

ANS: A Myocardial ischemia is a contraindication for ventilator weaning. The ST segment elevation is an indication that weaning should be postponed until further investigation and/or treatment for myocardial ischemia can be done. The other information will also be shared with the health care provider, but ventilator weaning can proceed when opioids are used for pain management, abnormal lung sounds are present, or enteral feedings are being used.

9. A client is receiving norepinephrine (Levophed) for shock. What assessment finding best indicates a therapeutic effect from this drug? a. Alert and oriented, answering questions b. Client denial of chest pain or chest pressure c. IV site without redness or swelling d. Urine output of 30 mL/hr for 2 hours

ANS: A Normal cognitive function is a good indicator that the client is receiving the benefits of norepinephrine. The brain is very sensitive to changes in oxygenation and perfusion. Norepinephrine can cause chest pain as an adverse reaction, so the absence of chest pain does not indicate therapeutic effect. The IV site is normal. The urine output is normal, but only minimally so.

The home health nurse is providing dietary recommendations to keep the immune system healthy. The patient demonstrates understanding by increasing intake of which foods? a.Eggs and beans b.Celery and water c.Pasta and bread d.Olive oil and peanuts

ANS: A Protein stores must be kept at an adequate level in order to produce antibodies, thus boosting the immune system. Eggs and beans are a good source of protein. Increasing intake of celery and water increases fluid. Pasta and bread are carbohydrate-rich foods. Olive oil and peanuts feature unsaturated fats. Fluids, carbohydrates, and unsaturated fats will not enhance the immune system. PTS: 1 DIF: Cognitive Level: Application REF: 103 OBJ: 8 (theory) TOP: Nutrition KEY: Nursing Process Step: Implementation MSC: NCLEX: Health Promotion and Maintenance: Prevention and Early Detection of Disease

The nurse provides discharge teaching about antibiotic therapy. Which statement indicates that the patient requires additional teaching? a."I should wait 3 days after my symptoms resolve before stopping my antibiotic." b."I should try to take my medication as evenly spaced apart as possible." c."If I start feeling worse, I should call my health care provider." d."I should not share my medication with anyone."

ANS: A The antibiotic should be taken until it is completely gone in order to ensure the infection has been adequately treated. Antibiotics are more effective if spaced evenly apart when taken. The patient should continue to improve if therapy is effective, so the health care provider should be notified if symptoms are not improving. Patients should never share any type of prescribed medication. PTS: 1 DIF: Cognitive Level: Application REF: 113, Table 6-7 OBJ: 8 (theory) TOP: Antimicrobial Therapy KEY: Nursing Process Step: Evaluation MSC: NCLEX: Health Promotion and Maintenance

Which assessment finding obtained by the nurse when caring for a patient with a right radial arterial line indicates a need for the nurse to take immediate action? a. The right hand is cooler than the left hand. b. The mean arterial pressure (MAP) is 77 mm Hg. c. The system is delivering 3 mL of flush solution per hour. d. The flush bag and tubing were last changed 3 days previously.

ANS: A The change in temperature of the left hand suggests that blood flow to the left hand is impaired. The flush system needs to be changed every 96 hours. A mean arterial pressure (MAP) of 75 mm Hg is normal. Flush systems for hemodynamic monitoring are set up to deliver 3 to 6 mL/hour of flush solution.

During change-of-shift report, the nurse is told that a patient has been admitted with dehydration and hypotension after having vomiting and diarrhea for 4 days. Which finding is most important for the nurse to report to the health care provider? a. New onset of confusion c. Heart rate 112 beats/min b. Decreased bowel sounds d. Pale, cool, and dry extremities

ANS: A The changes in mental status are indicative that the patient is in the progressive stage of shock and that rapid intervention is needed to prevent further deterioration. The other information is consistent with compensatory shock.

19. During change-of-shift report, the nurse is told that a patient has been admitted with dehydration and hypotension after having vomiting and diarrhea for 4 days. Which finding ismost important for the nurse to report to the health care provider? a. New onset of confusion b. Heart rate 112 beats/minute c. Decreased bowel sounds d. Pale, cool, and dry extremities

ANS: A The changes in mental status are indicative that the patient is in the progressive stage of shock and that rapid intervention is needed to prevent further deterioration. The other information is consistent with compensatory shock. DIF: Cognitive Level: Apply (application) REF: 1639 OBJ: Special Questions: Prioritization TOP: Nursing Process: Assessment MSC: NCLEX: Physiological Integrity

Which data collected by the nurse caring for a patient who has cardiogenic shock indicate that the patient may be developing multiple organ dysfunction syndrome (MODS)? a. The patient's serum creatinine level is elevated. b. The patient complains of intermittent chest pressure. c. The patient's extremities are cool and pulses are weak. d. The patient has bilateral crackles throughout lung fields.

ANS: A The elevated serum creatinine level indicates that the patient has renal failure as well as heart failure. The crackles, chest pressure, and cool extremities are all symptoms consistent with the patient's diagnosis of cardiogenic shock.

14. Which data collected by the nurse caring for a patient who has cardiogenic shock indicate that the patient may be developing multiple organ dysfunction syndrome (MODS)? a. The patient's serum creatinine level is elevated. b. The patient complains of intermittent chest pressure. c. The patient's extremities are cool and pulses are weak. d. The patient has bilateral crackles throughout lung fields.

ANS: A The elevated serum creatinine level indicates that the patient has renal failure as well as heart failure. The crackles, chest pressure, and cool extremities are all consistent with the patient's diagnosis of cardiogenic shock. DIF: Cognitive Level: Apply (application) REF: 1649 | 1633 TOP: Nursing Process: Assessment MSC: NCLEX: Physiological Integrity

A patient with respiratory failure has arterial pressure-based cardiac output (APCO) monitoring and is receiving mechanical ventilation with peak end-expiratory pressure (PEEP) of 12 cm H2O. Which information indicates that a change in the ventilator settings may be required? a. The arterial pressure is 90/46. b. The heart rate is 58 beats/minute. c. The stroke volume is increased. d. The stroke volume variation is 12%.

ANS: A The hypotension suggests that the high intrathoracic pressure caused by the PEEP may be decreasing venous return and (potentially) cardiac output. The other assessment data would not be a direct result of PEEP and mechanical ventilation.

18. A patient is admitted to the emergency department (ED) for shock of unknown etiology. Thefirst action by the nurse should be to a. administer oxygen. b. obtain a 12-lead electrocardiogram (ECG). c. obtain the blood pressure. d. check the level of consciousness.

ANS: A The initial actions of the nurse are focused on the ABCs—airway, breathing, and circulation—and administration of oxygen should be done first. The other actions should be accomplished as rapidly as possible after oxygen administration. DIF: Cognitive Level: Apply (application) REF: 1641 OBJ: Special Questions: Prioritization TOP: Nursing Process: Implementation MSC: NCLEX: Physiological Integrity

A patient who is orally intubated and receiving mechanical ventilation is anxious and is "fighting" the ventilator. Which action should the nurse take next? a. Verbally coach the patient to breathe with the ventilator. b. Sedate the patient with the ordered PRN lorazepam (Ativan). c. Manually ventilate the patient with a bag-valve-mask device. d. Increase the rate for the ordered propofol (Diprivan) infusion.

ANS: A The initial response by the nurse should be to try to decrease the patient's anxiety by coaching the patient about how to coordinate respirations with the ventilator. The other actions may also be helpful if the verbal coaching is ineffective in reducing the patient's anxiety.

13. A client in shock is apprehensive and slightly confused. What action by the nurse is best? a. Offer to remain with the client for awhile. b. Prepare to administer antianxiety medication. c. Raise all four siderails on the client's bed. d. Tell the client everything possible is being done.

ANS: A The nurse's presence will be best to reassure this client. Antianxiety medication is not warranted as this will lower the client's blood pressure. Using all four siderails on a hospital bed is considered a restraint in most facilities, although the nurse should ensure the client's safety. Telling a confused client that everything is being done is not the most helpful response.

11. A client has been brought to the emergency department after being shot multiple times. What action should the nurse perform first? a. Apply personal protective equipment. b. Notify local law enforcement officials. c. Obtain "universal" donor blood. d. Prepare the client for emergency surgery.

ANS: A The nurse's priority is to care for the client. Since the client has gunshot wounds and is bleeding, the nurse applies personal protective equipment (i.e., gloves) prior to care. This takes priority over calling law enforcement. Requesting blood bank products can be delegated. The nurse may or may not have to prepare the client for emergency surgery.

What should be the major consideration when selecting toys for a child with an intellectual or developmental disability? a. Safety b. Age appropriateness c. Ability to provide exercise d. Ability to teach useful skills

ANS: A Feedback A Safety is the primary concern in selecting recreational and exercise activities for all children. This is especially true for children who are intellectually disabled. B Age appropriateness should be considered in the selection of toys, but safety is of paramount importance. C Ability to provide exercise should be considered in the selection of toys, but safety is of paramount importance. D Ability to teach useful skills should be considered in the selection of toys, but safety is of paramount importance.

Parents have learned that their 6-year-old child has autism. The nurse may help the parents to cope by explaining that the child may a. Have an extremely developed skill in a particular area b. Outgrow the condition by early adulthood c. Have average social skills d. Have age-appropriate language skills

ANS: A Feedback A Some children with autism have an extremely developed skill in a particular area such as mathematics or music. B No evidence supports that autism is outgrown. C Autistic children have abnormal ways of relating to people (social skills). D Speech and language skills are usually delayed in autistic children.

Which preventive actions by the nurse will help limit the development of systemic inflammatory response syndrome (SIRS) in patients admitted to the hospital (select all that apply)? a. Ambulate postoperative patients as soon as possible after surgery. b. Use aseptic technique when manipulating invasive lines or devices. c. Remove indwelling urinary catheters as soon as possible after surgery. d. Administer prescribed antibiotics within 1 hour for patients with possible sepsis. e. Advocate for parenteral nutrition for patients who cannot take in adequate calories.

ANS: A, B, C, D Because sepsis is the most frequent etiology for SIRS, measures to avoid infection such as removing indwelling urinary catheters as soon as possible, use of aseptic technique, and early ambulation should be included in the plan of care. Adequate nutrition is important in preventing SIRS. Enteral, rather than parenteral, nutrition is preferred when patients are unable to take oral feedings because enteral nutrition helps maintain the integrity of the intestine, thus decreasing infection risk. Antibiotics should be given within 1 hour after being prescribed to decrease the risk of sepsis progressing to SIRS.

3. The nurse caring frequently for older adults in the hospital is aware of risk factors that place them at a higher risk for shock. For what factors would the nurse assess? (Select all that apply.) a. Altered mobility/immobility b. Decreased thirst response c. Diminished immune response d. Malnutrition e. Overhydration

ANS: A, B, C, D Immobility, decreased thirst response, diminished immune response, and malnutrition can place the older adult at higher risk of developing shock. Overhydration is not a common risk factor for shock.

The home health nurse in Wyoming gives instruction to an 80-year-old patient in the prevention of hypothermia when outdoors for long periods of time, which includes: (Select all that apply.) a. wearing multiple layers of clothing. b. wearing a snug-fitting hat. c. moving about briskly. d. drinking warm fluids from thermos. e. wearing gloves and earmuffs.

ANS: A, B, C, D, E All options listed are helpful in the prevention of hypothermia.

The nurse stresses that moving a victim of an automobile accident is necessary if there is: (Select all that apply.) a. pooled gasoline. b. oncoming traffic. c. submersion in snow. d. request from the victim to be moved. e. exposure to hot pavement.

ANS: A, B, C, E A victim request to be moved is not a valid reason to do so if the victim is safe.

2. Which preventive actions by the nurse will help limit the development of systemic inflammatory response syndrome (SIRS) in patients admitted to the hospital (select all that apply)? a. Use aseptic technique when caring for invasive lines or devices. b. Ambulate postoperative patients as soon as possible after surgery. c. Remove indwelling urinary catheters as soon as possible after surgery. d. Advocate for parenteral nutrition for patients who cannot take oral feedings. e. Administer prescribed antibiotics within 1 hour for patients with possible sepsis.

ANS: A, B, C, E Because sepsis is the most frequent etiology for SIRS, measures to avoid infection such as removing indwelling urinary catheters as soon as possible, use of aseptic technique, and early ambulation should be included in the plan of care. Adequate nutrition is important in preventing SIRS. Enteral, rather than parenteral, nutrition is preferred when patients are unable to take oral feedings because enteral nutrition helps maintain the integrity of the intestine, thus decreasing infection risk. Antibiotics should be administered within 1 hour after being prescribed to decrease the risk of sepsis progressing to SIRS. DIF: Cognitive Level: Analyze (analysis) REF: 1649 TOP: Nursing Process: Planning MSC: NCLEX: Physiological Integrity

You are the nurse assessing a 3-year-old child who has characteristics of autism. Which observed behaviors are associated with autism? Select all that apply. a. The child flicks the light in the examination room on and off repetitiously. b. The child has a flat affect. c. The child demonstrates imitation and gesturing skills. d. Mother reports the child has no interest in playing with other children. e. The child is able to make eye contact.

ANS: A, B, D Feedback Correct Self-stimulation is common and generally involves repetition of a sensory stimulus. Autistic children generally show a fixed, unchanging response to a particular stimulus. Autistic children generally play alone or involve others only as mere objects. Incorrect Autistic children lack imitative skills. These children lack social ability and make poor eye contact.

A patient with suspected neurogenic shock after a diving accident has arrived in the emergency department. A cervical collar is in place. Which actions should the nurse take (select all that apply)? a. Prepare to administer atropine IV. b. Obtain baseline body temperature. c. Infuse large volumes of lactated Ringer's solution. d. Provide high-flow O2 (100%) by nonrebreather mask. e. Prepare for emergent intubation and mechanical ventilation.

ANS: A, B, D, E All of the actions are appropriate except to give large volumes of lactated Ringer's solution. The patient with neurogenic shock usually has a normal blood volume, and it is important not to volume overload the patient. In addition, lactated Ringer's solution is used cautiously in all shock situations because an ischemic liver cannot convert lactate to bicarbonate.

MULTIPLE RESPONSE 1. A patient with suspected neurogenic shock after a diving accident has arrived in the emergency department. A cervical collar is in place. Which actions should the nurse take (select all that apply)? a. Prepare to administer atropine IV. b. Obtain baseline body temperature. c. Infuse large volumes of lactated Ringer's solution. d. Provide high-flow oxygen (100%) by non-rebreather mask. e. Prepare for emergent intubation and mechanical ventilation.

ANS: A, B, D, E All of the actions are appropriate except to give large volumes of lactated Ringer's solution. The patient with neurogenic shock usually has a normal blood volume, and it is important not to volume overload the patient. In addition, lactated Ringer's solution is used cautiously in all shock situations because the failing liver cannot convert lactate to bicarbonate. DIF: Cognitive Level: Apply (application) REF: 1646 TOP: Nursing Process: Implementation MSC: NCLEX: Physiological Integrity

The nurse instructs the nursing assistant in a long-term care facility regarding infection control measures. Which action(s) demonstrate(s) that the nursing assistant understands the nurse's teaching? (Select all that apply.) a.Assisting residents with hand hygiene before meals. b.Cleaning incontinent residents as soon as possible. c.Administering prescribed antibiotics during meals to residents who require assistance with feeding. d.Inspecting residents' skin for open areas during bathing. e.Assisting residents with hand hygiene after participating in group activities.

ANS: A, B, D, E It is important for the nursing assistant to assist residents with hand hygiene prior to meals and after participating in group activities in order to help prevent the spread of infection. Cleaning incontinent residents as soon as possible prevents skin breakdown, which may lead to infection. While bathing residents, the nursing assistant should monitor for signs of skin breakdown and report any areas to the nurse. Nursing assistants are not permitted to administer medications. PTS: 1 DIF: Cognitive Level: Application REF: 121 OBJ: 10 (clinical) TOP: Infection Control Measures KEY: Nursing Process Step: Evaluation MSC: NCLEX: Health Promotion and Maintenance

26. The nurse is working with a group of hikers who range in age from 25 to 35 years. They have been instructed on ways to prevent heatstroke. Which drink selection by a participant indicates the need for further education? (Select all that apply.) a. Clear carbonated soda b. Diet caffeinated cola c. Water d. Beer e. Sugar-sweetened energy drinks

ANS: A, B, D, E To aid in the prevention of heatstroke, the hiker should drink plenty of fluids that are nonalcoholic, caffeine free, and low in sugar content as the wrong fluids can increase fluid loss. Clear carbonated soda and sweetened energy drinks are high in sugar. Diet cola contains caffeine.

The nurse is aware that the treatment for frostbite includes: (Select all that apply.) a. chafing the hands and fingers gently to reestablish circulation. b. immersion of hands and feet in warm water. c. wrapping hands in mitten-like dressings to retain warmth. d. administering opioids to reduce pain. e. elevating affected limbs.

ANS: A, B, E Analgesia will be accomplished with nonsteroidal anti-inflammatory drugs as opioids decrease function and delay circulatory recovery. Fingers should be wrapped individually, not touching each other.

5. The nurse is caring for a client with suspected severe sepsis. What does the nurse prepare to do within 3 hours of the client being identified as being at risk? (Select all that apply.) a. Administer antibiotics. b. Draw serum lactate levels. c. Infuse vasopressors. d. Measure central venous pressure. e. Obtain blood cultures.

ANS: A, B, E Within the first 3 hours of suspecting severe sepsis, the nurse should draw (or facilitate) serum lactate levels, obtain blood cultures (or other cultures), and administer antibiotics (after the cultures have been obtained). Infusing vasopressors and measuring central venous pressure are actions that should occur within the first 6 hours.

A nurse is assessing a newborn for facial feature characteristics associated with fetal alcohol syndrome: Which characteristics should the nurse expect to assess? Select all that apply. a. Short palpebral fissures b. Smooth philtrum c. Low set ears d. Inner epicanthal folds e. Thin upper lip

ANS: A, B, E Feedback Correct: Infants with fetal alcohol syndrome may have characteristic facial features, including short palpebral fissures, a smooth philtrum (the vertical groove in the median portion of the upper lip), and a thin upper lip. Incorrect: Low set ears and inner epicanthal folds are associated with Down syndrome

The nurse explains that the older adult is prone to hypothermia because the older adult: (Select all that apply.) a. eats less. b. has more subcutaneous fat. c. has lower metabolism. d. has atherosclerosis. e. is less active.

ANS: A, C, D, E The older adult has less subcutaneous fat

4. A client is in the early stages of shock and is restless. What comfort measures does the nurse delegate to the nursing student? (Select all that apply.) a. Bringing the client warm blankets b. Giving the client hot tea to drink c. Massaging the client's painful legs d. Reorienting the client as needed e. Sitting with the client for reassurance

ANS: A, D, E The student can bring the client warm blankets, reorient the client as needed to decrease anxiety, and sit with the client for reassurance. The client should be NPO at this point, so hot tea is prohibited. Massaging the legs is not recommended as this can dislodge any clots present, which may lead to pulmonary embolism.

The nurse is obtaining a health history on a newly admitted patient. Which information alerts the nurse that the patient is at increased risk for developing an infection? (Select all that apply.) a.The patient reports having unprotected heterosexual sex in three previous relationships. b.The patient is employed as a biochemist in a hospital. c.The patient's income is considered middle-class level. d.The patient reports getting 4 to 5 hours of sleep per night. e.The patient is 21% over the suggested normal weight.

ANS: A, D, E This patient's lifestyle habits, insufficient sleep, and being obese increase the chance of developing an infection by the strain placed on the immune system. This patient's occupation and income level would not increase the risk for infection. PTS: 1 DIF: Cognitive Level: Application REF: 100, Box 6-1 OBJ: 10 (clinical) TOP: Risk Factors KEY: Nursing Process Step: Implementation MSC: NCLEX: Health Promotion and Maintenance COMPLETION

A nurse should plan to implement which interventions for a child admitted with inorganic failure to thrive? Select all that apply. a. Observation of parent-child interactions b. Assignment of different nurses to care for the child from day to day c. Use of 28 calorie per ounce concentrated formulas d. Administration of daily multivitamin supplements e. Role modeling appropriate adult-child interactions

ANS: A, D, E Feedback Correct: The nurse should plan to assess parent-child interactions when a child is admitted for nonorganic failure to thrive. The observations should include how the child is held and fed, how eye contact is initiated and maintained, and the facial expressions of both the child and the caregiver during interactions. Role modeling and teaching appropriate adult-child interactions (including holding, touching, and feeding the child) will facilitate appropriate parent-child relationships, enhance parents' confidence in caring for their child, and facilitate expression by the parents of realistic expectations based on the child's developmental needs. Daily multivitamin supplements with minerals are often prescribed to ensure that specific nutritional deficiencies do not occur in the course of rapid growth. The nursing staff assigned to care for the child should be consistent. Incorrect: Providing a consistent caregiver from the nursing staff increases trust and provides the child with an adult who anticipates his or her needs and who is able to role model child care to the parent. Caloric enrichment of food is essential, and formula may be concentrated in titrated amounts up to 24 calories per ounce. Greater concentrations can lead to diarrhea and dehydration.

12. A nurse is caring for several clients at risk for shock. Which laboratory value requires the nurse to communicate with the health care provider? a. Creatinine: 0.9 mg/dL b. Lactate: 6 mmol/L c. Sodium: 150 mEq/L d. White blood cell count: 11,000/mm3

ANS: B A lactate level of 6 mmol/L is high and is indicative of possible shock. A creatinine level of 0.9 mg/dL is normal. A sodium level of 150 mEq/L is high, but that is not related directly to shock. A white blood cell count of 11,000/mm3 is slightly high but is not as critical as the lactate level.

Following surgery for an abdominal aortic aneurysm, a patient's central venous pressure (CVP) monitor indicates low pressures. Which action is a priority for the nurse to take? a. Administer IV diuretic medications. b. Increase the IV fluid infusion per protocol. c. Document the CVP and continue to monitor. d. Elevate the head of the patient's bed to 45 degrees.

ANS: B A low CVP indicates hypovolemia and a need for an increase in the infusion rate. Diuretic administration will contribute to hypovolemia and elevation of the head may decrease cerebral perfusion. Documentation and continued monitoring is an inadequate response to the low CVP.

Norepinephrine has been prescribed for a patient who was admitted with dehydration and hypotension. Which patient data indicate that the nurse should consult with the health care provider before starting the norepinephrine? a. The patient is receiving low dose dopamine. b. The patient's central venous pressure is 3 mm Hg. c. The patient is in sinus tachycardia at 120 beats/min. d. The patient has had no urine output since being admitted.

ANS: B Adequate fluid administration is essential before giving vasopressors to patients with hypovolemic shock. The patient's low central venous pressure indicates a need for more volume replacement. The other patient data are not contraindications to norepinephrine administration.

8. A client arrives in the emergency department after being in a car crash with fatalities. The client has a nearly amputated leg that is bleeding profusely. What action by the nurse takes priority? a. Apply direct pressure to the bleeding. b. Ensure the client has a patent airway. c. Obtain consent for emergency surgery. d. Start two large-bore IV catheters.

ANS: B Airway is the priority, followed by breathing and circulation (IVs and direct pressure). Obtaining consent is done by the physician.

14. A client is being discharged home after a large myocardial infarction and subsequent coronary artery bypass grafting surgery. The client's sternal wound has not yet healed. What statement by the client most indicates a higher risk of developing sepsis after discharge? a. "All my friends and neighbors are planning a party for me." b. "I hope I can get my water turned back on when I get home." c. "I am going to have my daughter scoop the cat litter box." d. "My grandkids are so excited to have me coming home!"

ANS: B All these statements indicate a potential for leading to infection once the client gets back home. A large party might include individuals who are themselves ill and contagious. Having litter boxes in the home can expose the client to microbes that can lead to infection. Small children often have upper respiratory infections and poor hand hygiene that spread germs. However, the most worrisome statement is the lack of running water for handwashing and general hygiene and cleaning purposes.

24. After change-of-shift report in the progressive care unit, who should the nurse care for first? a. Patient who had an inferior myocardial infarction 2 days ago and has crackles in the lung bases b. Patient with suspected urosepsis who has new orders for urine and blood cultures and antibiotics c. Patient who had a T5 spinal cord injury 1 week ago and currently has a heart rate of 54 beats/minute d. Patient admitted with anaphylaxis 3 hours ago who now has clear lung sounds and a blood pressure of 108/58 mm Hg

ANS: B Antibiotics should be administered within the first hour for patients who have sepsis or suspected sepsis in order to prevent progression to systemic inflammatory response syndrome (SIRS) and septic shock. The data on the other patients indicate that they are more stable. Crackles heard only at the lung bases do not require immediate intervention in a patient who has had a myocardial infarction. Mild bradycardia does not usually require atropine in patients who have a spinal cord injury. The findings for the patient admitted with anaphylaxis indicate resolution of bronchospasm and hypotension. DIF: Cognitive Level: Analyze (analysis) REF: 1644 | 1646 OBJ: Special Questions: Prioritization; Multiple Patients TOP: Nursing Process: Assessment MSC: NCLEX: Safe and Effective Care Environment

After change-of-shift report in the progressive care unit, who should the nurse care for first? a. Patient who had an inferior myocardial infarction 2 days ago and has crackles in the lung bases b. Patient with suspected urosepsis who has new orders for urine and blood cultures and antibiotics c. Patient who had a T5 spinal cord injury 1 week ago and currently has a heart rate of 54 beats/minute d. Patient admitted with anaphylaxis 3 hours ago who now has clear lung sounds and a blood pressure of 108/58 mm Hg

ANS: B Antibiotics should be given within the first hour for patients who have sepsis or suspected sepsis in order to prevent progression to systemic inflammatory response syndrome and septic shock. The data on the other patients indicate that they are more stable. Crackles heard only at the lung bases do not require immediate intervention in a patient who has had a myocardial infarction. Mild bradycardia does not usually require atropine in patients who have a spinal cord injury. The findings for the patient admitted with anaphylaxis indicate resolution of bronchospasm and hypotension.

Which finding is the best indicator that the fluid resuscitation for a 90-kg patient with hypovolemic shock has been effective? a. Hemoglobin is within normal limits. b. Urine output is 65 mL over the past hour. c. Central venous pressure (CVP) is normal. d. Mean arterial pressure (MAP) is 72 mm Hg.

ANS: B Assessment of end organ perfusion, such as an adequate urine output, is the best indicator that fluid resuscitation has been successful. Urine output should be equal to or more than 0.5 mL/kg/hr. The hemoglobin level, CVP, and MAP are useful in determining the effects of fluid administration, but they are not as useful as data indicating good organ perfusion.

9. Which finding is the best indicator that the fluid resuscitation for a patient with hypovolemic shock has been effective? a. Hemoglobin is within normal limits. b. Urine output is 60 mL over the last hour. c. Central venous pressure (CVP) is normal. d. Mean arterial pressure (MAP) is 72 mm Hg.

ANS: B Assessment of end organ perfusion, such as an adequate urine output, is the best indicator that fluid resuscitation has been successful. The hemoglobin level, CVP, and MAP are useful in determining the effects of fluid administration, but they are not as useful as data indicating good organ perfusion. DIF: Cognitive Level: Apply (application) REF: 1642 TOP: Nursing Process: Evaluation MSC: NCLEX: Physiological Integrity

17. The nurse is caring for a patient who has septic shock. Which assessment finding is mostimportant for the nurse to report to the health care provider? a. Blood pressure (BP) 92/56 mm Hg b. Skin cool and clammy c. Oxygen saturation 92% d. Heart rate 118 beats/minute

ANS: B Because patients in the early stage of septic shock have warm and dry skin, the patient's cool and clammy skin indicates that shock is progressing. The other information will also be reported, but does not indicate deterioration of the patient's status. DIF: Cognitive Level: Apply (application) REF: 1638 OBJ: Special Questions: Prioritization TOP: Nursing Process: Assessment MSC: NCLEX: Physiological Integrity

Which intervention will the nurse include in the plan of care for a patient who has cardiogenic shock? a. Check temperature every 2 hours. b. Monitor breath sounds frequently. c. Maintain patient in supine position. d. Assess skin for flushing and itching.

ANS: B Because pulmonary congestion and dyspnea are characteristics of cardiogenic shock, the nurse should assess the breath sounds frequently. The head of the bed is usually elevated to decrease dyspnea in patients with cardiogenic shock. Elevated temperature and flushing or itching of the skin are not typical of cardiogenic shock.

Which finding about a patient who is receiving vasopressin to treat septic shock indicates an immediate need for the nurse to report the finding to the health care provider? a. The patient's urine output is 18 mL/hr. b. The patient is complaining of chest pain. c. The patient's peripheral pulses are weak. d. The patient's heart rate is 110 beats/minute.

ANS: B Because vasopressin is a potent vasoconstrictor, it may decrease coronary artery perfusion. The other information is consistent with the patient's diagnosis, and should be reported to the health care provider but does not indicate an immediate need for a change in therapy.

A 68-year-old patient has been in the intensive care unit for 4 days and has a nursing diagnosis of disturbed sensory perception related to sleep deprivation. Which action should the nurse include in the plan of care? a. Administer prescribed sedatives or opioids at bedtime to promote sleep. b. Cluster nursing activities so that the patient has uninterrupted rest periods. c. Silence the alarms on the cardiac monitors to allow 30- to 40-minute naps. d. Eliminate assessments between 0100 and 0600 to allow uninterrupted sleep.

ANS: B Clustering nursing activities and providing uninterrupted rest periods will minimize sleep-cycle disruption. Sedative and opioid medications tend to decrease the amount of rapid eye movement (REM) sleep and can contribute to sleep disturbance and disturbed sensory perception. Silencing the alarms on the cardiac monitors would be unsafe in a critically ill patient, as would discontinuing assessments during the night.

The central venous oxygen saturation (ScvO2) is decreasing in a patient who has severe pancreatitis. To determine the possible cause of the decreased ScvO2, the nurse assesses the patient's a. lipase. b. temperature. c. urinary output. d. body mass index.

ANS: B Elevated temperature increases metabolic demands and oxygen use by tissues, resulting in a drop in oxygen saturation of central venous blood. Information about the patient's body mass index, urinary output, and lipase will not help in determining the cause of the patient's drop in ScvO2.

22. The following interventions are ordered by the health care provider for a patient who has respiratory distress and syncope after eating strawberries. Which will the nurse complete first? a. Start a normal saline infusion. b. Give epinephrine (Adrenalin). c. Start continuous ECG monitoring. d. Give diphenhydramine (Benadryl).

ANS: B Epinephrine rapidly causes peripheral vasoconstriction, dilates the bronchi, and blocks the effects of histamine and reverses the vasodilation, bronchoconstriction, and histamine release that cause the symptoms of anaphylaxis. The other interventions are also appropriate but would not be the first ones completed. DIF: Cognitive Level: Apply (application) REF: 1645 OBJ: Special Questions: Prioritization TOP: Nursing Process: Implementation MSC: NCLEX: Physiological Integrity

The intensive care unit (ICU) nurse educator will determine that teaching about arterial pressure monitoring for a new staff nurse has been effective when the nurse a. balances and calibrates the monitoring equipment every 2 hours. b. positions the zero-reference stopcock line level with the phlebostatic axis. c. ensures that the patient is supine with the head of the bed flat for all readings. d. rechecks the location of the phlebostatic axis when changing the patient's position.

ANS: B For accurate measurement of pressures, the zero-reference level should be at the phlebostatic axis. There is no need to rebalance and recalibrate monitoring equipment hourly. Accurate hemodynamic readings are possible with the patient's head raised to 45 degrees or in the prone position. The anatomic position of the phlebostatic axis does not change when patients are repositioned.

A frustrated patient with a fungal infection complains, "Why is the infection taking so long to heal?" Which response is most appropriate? a."Fungal infections are essentially incurable. "b."Fungi form spores, which make them difficult to kill." c."Fungi can be considered natural flora and are protected by the body." d."Fungi can alter the patient's DNA and RNA."

ANS: B Fungi are capable of forming spores, which makes them resistant to antifungal agents. PTS: 1 DIF: Cognitive Level: Comprehension REF: 102 OBJ: 1 (theory) TOP: Fungi KEY: Nursing Process Step: Implementation MSC: NCLEX: Safe, Effective Care Environment: Safety and Infection Control

The nurse is providing infection control teaching to a group of patients. Which statement demonstrates that the patient understands the nurse's teaching? a."I should take an antibiotic at the first sign of an infection." b."Hand hygiene is one of the most effective ways I can prevent the spread of infection." c."Vaccinations only prevent a disease from becoming severe." d."If I eat a nutritious diet, it will be difficult for me to get an infection."

ANS: B Hand hygiene is the most effective single act that can reduce the spread of disease. Antibiotics should not be taken at the first sign of infection, especially if the infection is caused by a virus; vaccinations can also prevent diseases from occurring; a nutritious diet is only one component in the prevention of infection. PTS: 1 DIF: Cognitive Level: Application REF: 107 OBJ: 2 (theory) TOP: Hand Hygiene KEY: Nursing Process Step: Evaluation MSC: NCLEX: Health Promotion and Maintenance

1. A student is caring for a client who suffered massive blood loss after trauma. How does the student correlate the blood loss with the client's mean arterial pressure (MAP)? a. It causes vasoconstriction and increased MAP. b. Lower blood volume lowers MAP. c. There is no direct correlation to MAP. d. It raises cardiac output and MAP.

ANS: B Lower blood volume will decrease MAP. The other answers are not accurate.

The nurse explains to the patient who is using Prilosec (a proton pump inhibitor) that the drug reduces the amount of which natural protector in the stomach lining? a.Lactic acid b.Lysozyme c.Cilia d.Fatty acids

ANS: B Lysozyme is found in the lining of the stomach and in the stomach acids. PTS: 1 DIF: Cognitive Level: Knowledge REF: 102 OBJ: 2 (theory) TOP: Chemical Barrier KEY: Nursing Process Step: Implementation MSC: NCLEX: Physiological Integrity: Pharmacological Therapies

A patient with massive trauma and possible spinal cord injury is admitted to the emergency department (ED). Which assessment finding by the nurse will help confirm a diagnosis of neurogenic shock? a. Inspiratory crackles c. Cool, clammy extremities b. Heart rate 45 beats/min d. Temperature 101.2°F (38.4°C)

ANS: B Neurogenic shock is characterized by hypotension and bradycardia. The other findings would be more consistent with other types of shock.

The nurse is admitting a patient who has a neck fracture at the C6 level to the intensive care unit. Which assessment findings indicate neurogenic shock? a. Involuntary and spastic movement b. Hypotension and warm extremities c. Hyperactive reflexes below the injury d. Lack of sensation or movement below the injury

ANS: B Neurogenic shock is characterized by hypotension, bradycardia, and vasodilation leading to warm skin temperature. Spasticity and hyperactive reflexes do not occur at this stage of spinal cord injury. Lack of movement and sensation indicate spinal cord injury but not neurogenic shock.

The nurse has arrived on the scene of an accident. The victim is conscious and has a large bleeding laceration on his thigh. After the nurse uses an available towel to provide compression to the wound, what action should be performed next? a. The nurse should turn the patient to his left side. b. The nurse should elevate the patient's affected leg. c. The nurse should bend the affected leg at the knee. d. The nurse should encourage the patient to perform leg pump exercises.

ANS: B Once the bleeding has stopped, a compression dressing and bulky bandage are applied and left in place to prevent disturbing clots gently. Elevating and immobilizing the injured part will help to control bleeding.

7. A nurse works at a community center for older adults. What self-management measure can the nurse teach the clients to prevent shock? a. Do not get dehydrated in warm weather. b. Drink fluids on a regular schedule. c. Seek attention for any lacerations. d. Take medications as prescribed.

ANS: B Preventing dehydration in older adults is important because the age-related decrease in the thirst mechanism makes them prone to dehydration. Having older adults drink fluids on a regular schedule will help keep them hydrated without the influence of thirst (or lack of thirst). Telling clients not to get dehydrated is important, but not the best answer because it doesn't give them the tools to prevent it from occurring. Older adults should seek attention for lacerations, but this is not as important an issue as staying hydrated. Taking medications as prescribed may or may not be related to hydration.

When caring for a patient who has an arterial catheter in the left radial artery for arterial pressure-based cardiac output (APCO) monitoring, which information obtained by the nurse is most important to report to the health care provider? a. The patient has a positive Allen test. b. There is redness at the catheter insertion site. c. The mean arterial pressure (MAP) is 86 mm Hg. d. The dicrotic notch is visible in the arterial waveform.

ANS: B Redness at the catheter insertion site indicates possible infection. The Allen test is performed before arterial line insertion, and a positive test indicates normal ulnar artery perfusion. A MAP of 86 is normal and the dicrotic notch is normally present on the arterial waveform.

While family members are visiting, a patient has a respiratory arrest and is being resuscitated. Which action by the nurse is best? a. Tell the family members that watching the resuscitation will be very stressful. b. Ask family members if they wish to remain in the room during the resuscitation. c. Take the family members quickly out of the patient room and remain with them. d. Assign a staff member to wait with family members just outside the patient room.

ANS: B Research indicates that family members want the option of remaining in the room during procedures such as cardiopulmonary resuscitation (CPR) and that this decreases anxiety and facilitates grieving. The other options may be appropriate if the family decides not to remain with the patient.

2. A nurse is caring for a client after surgery. The client's respiratory rate has increased from 12 to 18 breaths/min and the pulse rate increased from 86 to 98 beats/min since they were last assessed 4 hours ago. What action by the nurse is best? a. Ask if the client needs pain medication. b. Assess the client's tissue perfusion further. c. Document the findings in the client's chart. d. Increase the rate of the client's IV infusion.

ANS: B Signs of the earliest stage of shock are subtle and may manifest in slight increases in heart rate, respiratory rate, or blood pressure. Even though these readings are not out of the normal range, the nurse should conduct a thorough assessment of the client, focusing on indicators of perfusion. The client may need pain medication, but this is not the priority at this time. Documentation should be done thoroughly but is not the priority either. The nurse should not increase the rate of the IV infusion without an order.

10. Which intervention will the nurse include in the plan of care for a patient who has cardiogenic shock? a. Check temperature every 2 hours. b. Monitor breath sounds frequently. c. Maintain patient in supine position. d. Assess skin for flushing and itching.

ANS: B Since pulmonary congestion and dyspnea are characteristics of cardiogenic shock, the nurse should assess the breath sounds frequently. The head of the bed is usually elevated to decrease dyspnea in patients with cardiogenic shock. Elevated temperature and flushing or itching of the skin are not typical of cardiogenic shock. DIF: Cognitive Level: Apply (application) REF: 1633 TOP: Nursing Process: Implementation MSC: NCLEX: Physiological Integrity

Which hemodynamic parameter is most appropriate for the nurse to monitor to determine the effectiveness of medications given to a patient to reduce left ventricular afterload? a. Mean arterial pressure (MAP) b. Systemic vascular resistance (SVR) c. Pulmonary vascular resistance (PVR) d. Pulmonary artery wedge pressure (PAWP)

ANS: B Systemic vascular resistance reflects the resistance to ventricular ejection, or afterload. The other parameters will be monitored, but do not reflect afterload as directly.

The nurse is teaching a CPR class. During the class the nurse correctly includes which statement when discussing the Good Samaritan Law? a. The Good Samaritan Law protects only medical professionals from liability. b. The Good Samaritan Law protects all people from liability. c. The Good Samaritan Law limits the liability of a medical professional. d. The Good Samaritan Law defines specific situations in which no liability will occur.

ANS: B The Good Samaritan Law is designed to protect passersby who render first aid so they will not be held liable for the outcome of emergency care. Individuals who choose to render care will be held to the standard consistent with their training. The law is not limited to medical personnel. It does not address limitations of liability. There is no definition of specific scenarios protected by the law.

A patient with cardiogenic shock has the following vital signs: BP 102/50, pulse 128, respirations 28. The pulmonary artery wedge pressure (PAWP) is increased, and cardiac output is low. The nurse will anticipate an order for which medication? a. 5% albumin infusion c. epinephrine (Adrenalin) drip b. furosemide (Lasix) IV d. hydrocortisone (Solu-Cortef)

ANS: B The PAWP indicates that the patient's preload is elevated, and furosemide is indicated to reduce the preload and improve cardiac output. Epinephrine would further increase the heart rate and myocardial oxygen demand. 5% albumin would also increase the PAWP. Hydrocortisone might be considered for septic or anaphylactic shock.

7. A patient with cardiogenic shock has the following vital signs: BP 102/50, pulse 128, respirations 28. The pulmonary artery wedge pressure (PAWP) is increased and cardiac output is low. The nurse will anticipate an order for which medication? a. 5% human albumin b. Furosemide (Lasix) IV c. Epinephrine (Adrenalin) drip d. Hydrocortisone (Solu-Cortef)

ANS: B The PAWP indicates that the patient's preload is elevated, and furosemide is indicated to reduce the preload and improve cardiac output. Epinephrine would further increase heart rate and myocardial oxygen demand. 5% human albumin would also increase the PAWP. Hydrocortisone might be considered for septic or anaphylactic shock. DIF: Cognitive Level: Apply (application) REF: 1645 TOP: Nursing Process: Planning MSC: NCLEX: Physiological Integrity

A deliveryman comes to the emergency department with dog bites on his legs. He states that the dog ran away after the attack and could not be identified. After treatment of the bites, the patient should immediately: a. notify Animal Control. b. receive immune globulin for passive immunity. c. receive the first of five rabies vaccination injections to be continued over the next several weeks. d. go home and elevate his legs.

ANS: B The administration of immune globulin will build up his immediate defenses. As a deliveryman, he would be considered to be in a high-risk group for animal bites and should be advised to acquire the vaccine, but the vaccine will not be of any use to him at this point.

The nurse educator is evaluating the performance of a new registered nurse (RN) who is providing care to a patient who is receiving mechanical ventilation with 15 cm H2O of peak end-expiratory pressure (PEEP). Which action indicates that the new RN is safe? a. The RN plans to suction the patient every 1 to 2 hours. b. The RN uses a closed-suction technique to suction the patient. c. The RN tapes connection between the ventilator tubing and the ET. d. The RN changes the ventilator circuit tubing routinely every 48 hours.

ANS: B The closed-suction technique is used when patients require high levels of PEEP (>10 cm H2O) to prevent the loss of PEEP that occurs when disconnecting the patient from the ventilator. Suctioning should not be scheduled routinely, but it should be done only when patient assessment data indicate the need for suctioning. Taping connections between the ET and the ventilator tubing would restrict the ability of the tubing to swivel in response to patient repositioning. Ventilator tubing changes increase the risk for ventilator-associated pneumonia (VAP) and are not indicated routinely.

A patient who has been involved in a motor vehicle crash arrives in the emergency department (ED) with cool, clammy skin; tachycardia; and hypotension. Which intervention ordered by the health care provider should the nurse implement first? a. Insert two large-bore IV catheters. b. Provide O2 at 100% per non-rebreather mask. c. Draw blood to type and crossmatch for transfusions. d. Initiate continuous electrocardiogram (ECG) monitoring.

ANS: B The first priority in the initial management of shock is maintenance of the airway and ventilation. ECG monitoring, insertion of IV catheters, and obtaining blood for transfusions should also be rapidly accomplished but only after actions to maximize O2 delivery have been implemented.

The first responder to an automobile accident finds a victim with a sucking chest wound. The responder should: a. tightly bind the injury with a folded magazine and the patient's belt. b. place a plastic sandwich bag over the wound and tape on three sides to make a flutter dressing. c. turn the patient to the affected side and instruct the patient to deep breathe. d. place the patient's hand over the wound and tell the patient to press down.

ANS: B The flutter dressing will allow the air to leave the pleural space, but not allow any more air in. The collapsed lung will begin to reexpand.

When evaluating a patient with a central venous catheter, the nurse observes that the insertion site is red and tender to touch and the patient's temperature is 101.8° F. What should the nurse plan to do next? a. Give analgesics and antibiotics as ordered. b. Discontinue the catheter and culture the tip. c. Change the flush system and monitor the site. d. Check the site more frequently for any swelling.

ANS: B The information indicates that the patient has a local and systemic infection caused by the catheter, and the catheter should be discontinued. Changing the flush system, giving analgesics, and continued monitoring will not help prevent or treat the infection. Administration of antibiotics is appropriate, but the line should still be discontinued to avoid further complications such as endocarditis.

A nurse is caring for a patient whose hemodynamic monitoring indicates a blood pressure of 92/54 mm Hg, a pulse of 64 beats/min, and an elevated pulmonary artery wedge pressure (PAWP). Which intervention ordered by the health care provider should the nurse question? a. Elevate head of bed to 30 degrees. b. Infuse normal saline at 250 mL/hr. c. Hold nitroprusside if systolic BP is less than 90 mm Hg. d. Titrate dobutamine to keep systolic BP is greater than 90 mm Hg.

ANS: B The patient's elevated PAWP indicates volume excess in relation to cardiac pumping ability, consistent with cardiogenic shock. A saline infusion at 250 mL/hr will exacerbate the volume excess. The other actions will help to improve cardiac output, which should lower the PAWP and may raise the BP.

An 81-year-old patient who has been in the intensive care unit (ICU) for a week is now stable and transfer to the progressive care unit is planned. On rounds, the nurse notices that the patient has new onset confusion. The nurse will plan to a. give PRN lorazepam (Ativan) and cancel the transfer. b. inform the receiving nurse and then transfer the patient. c. notify the health care provider and postpone the transfer. d. obtain an order for restraints as needed and transfer the patient.

ANS: B The patient's history and symptoms most likely indicate delirium associated with the sleep deprivation and sensory overload in the ICU environment. Informing the receiving nurse and transferring the patient is appropriate. Postponing the transfer is likely to prolong the delirium. Benzodiazepines and restraints contribute to delirium and agitation.

The patient complains of the unsightly swelling of her lip at the site of an infection. The nurse explains that the swelling is part of the inflammatory response and performs which action? a.Stores blood b.Acts as a compression wall c.Provides an antibody reservoir d.Produces leukocytes

ANS: B The swelling of the inflammatory response acts as a compression wall to delay the spread of harmful agents to the rest of the body. PTS: 1 DIF: Cognitive Level: Comprehension REF: 106 OBJ: 2 (theory) TOP: Inflammatory Response KEY: Nursing Process Step: Implementation MSC: NCLEX: Health Promotion and Maintenance

4. A nurse is caring for a client after surgery who is restless and apprehensive. The unlicensed assistive personnel (UAP) reports the vital signs and the nurse sees they are only slightly different from previous readings. What action does the nurse delegate next to the UAP? a. Assess the client for pain or discomfort. b. Measure urine output from the catheter. c. Reposition the client to the unaffected side. d. Stay with the client and reassure him or her.

ANS: B Urine output changes are a sensitive early indicator of shock. The nurse should delegate emptying the urinary catheter and measuring output to the UAP as a baseline for hourly urine output measurements. The UAP cannot assess for pain. Repositioning may or may not be effective for decreasing restlessness, but does not take priority over physical assessments. Reassurance is a therapeutic nursing action, but the nurse needs to do more in this situation.

5. After receiving 2 L of normal saline, the central venous pressure for a patient who has septic shock is 10 mm Hg, but the blood pressure is still 82/40 mm Hg. The nurse will anticipate an order for a. nitroglycerine (Tridil). b. norepinephrine (Levophed). c. sodium nitroprusside (Nipride). d. methylprednisolone (Solu-Medrol).

ANS: B When fluid resuscitation is unsuccessful, vasopressor drugs are administered to increase the systemic vascular resistance (SVR) and blood pressure, and improve tissue perfusion. Nitroglycerin would decrease the preload and further drop cardiac output and BP. Methylprednisolone (Solu-Medrol) is considered if blood pressure does not respond first to fluids and vasopressors. Nitroprusside is an arterial vasodilator and would further decrease SVR. DIF: Cognitive Level: Apply (application) REF: 1643 TOP: Nursing Process: Planning MSC: NCLEX: Physiological Integrity

A nurse is giving a parent information about autism. Which statement made by the parent indicates understanding of the teaching? a. Autism is characterized by periods of remission and exacerbation. b. The onset of autism usually occurs before 3 years of age. c. Children with autism have imitation and gesturing skills. d. Autism can be treated effectively with medication.

ANS: B Feedback A Autism does not have periods of remission and exacerbation. B The onset of autism usually occurs before 3 years of age. C Autistic children lack imitative skills. D Medications are of limited use in children with autism.

A parent asks the nurse why a developmental assessment is being conducted for a child during a routine well-child visit. The nurse answers based on the knowledge that routine developmental assessments during well-child visits are a. Not necessary unless the parents request them b. The best method for early detection of cognitive disorders c. Frightening to parents and children and should be avoided d. Valuable in measuring intelligence in children

ANS: B Feedback A Developmental assessment is a component of all well-child examinations. B Early detection of cognitive disorders can be facilitated through assessment of development at each well-child examination. C Developmental assessments are not frightening when the parent and child are educated about the purpose of the assessment. D Developmental assessments are not intended to measure intelligence.

Developmental delays, self-injury, fecal smearing, and severe temper tantrums in a preschool child are symptoms of a. Down syndrome b. Intellectual disability c. Psychosocial deprivation d. Separation anxiety

ANS: B Feedback A Down syndrome is often identified at birth by characteristic facial and head features, such as brachycephaly (disproportionate shortness of the head); flat profile; inner epicanthal folds; wide, flat nasal bridge; narrow, high-arched palate; protruding tongue; and small, short ears, which may be low set. Although intellectual impairment may be present, the symptoms listed are not the primary ones expected in the diagnosis of Down syndrome. B These are symptoms of intellectual disability. C Psychosocial deprivation may be a cause of mild intellectual disability. The symptoms listed are characteristic of severe intellectual disability. D Symptoms of separation anxiety include protest, despair, and detachment.

A child with autism hospitalized with asthma. The nurse should plan care so that the a. Parents' expectations are met. b. Child's routine habits and preferences are maintained. c. Child is supported through the autistic crisis. d. Parents need not be at the hospital.

ANS: B Feedback A Focus of care is on the child's needs rather than on the parent's desires. B Children with autism are often unable to tolerate even slight changes in routine. The child's routine habits and preferences are important to maintain. C Autism is a life-long condition. D The presence of the parents is almost always required when an autistic child is hospitalized.

A parent whose child has been diagnosed with a cognitive deficit should be counseled that intellectual impairment a. Is usually due to a genetic defect b. May be caused by a variety of factors c. Is rarely due to first trimester events d. Is usually caused by parental intellectual impairment

ANS: B Feedback A Only a small percentage of children with intellectual impairment are affected by a genetic defect. B There are a multitude of causes for intellectual impairment. In most cases, a specific cause has not been identified. C One third of children with intellectual impairment are affected by first trimester events. D Intellectual impairment can be transmitted to a child only if the parent has a genetic disorder.

A newborn assessment shows separated sagittal suture, oblique palpebral fissures, depressed nasal bridge, protruding tongue, and transverse palmar creases. These findings are most suggestive of a. Microcephaly b. Down syndrome c. Cerebral palsy d. Fragile X syndrome

ANS: B Feedback A The infant with microcephaly has a small head. B These are characteristics associated with Down syndrome. C Cerebral palsy is a diagnosis not usually made at birth. No characteristic physical signs are present. D The infant with fragile X syndrome has increased head circumference; long, wide, and/or protruding ears; long, narrow face with prominent jaw; hypotonia; and high arched palate.

The nurse counsels a group of young track athletes that to prevent heatstroke they should: (Select all that apply.) a. drink plenty of fluids with high sugar content. b. wear lightweight, loose clothing. c. practice in the early morning. d. rest frequently in cool places. e. wear dark-colored clothing to block sun rays.

ANS: B, C, D Liquids should be nonalcoholic, noncaffeinated, and low sugar as liquids with alcohol, caffeine, and sugar increase dehydration. Light-colored clothing should be worn as dark colors absorb heat.

The nurse is caring for a patient suspected of having heatstroke. Which findings are consistent with this diagnosis? (Select all that apply.) a. Bradycardia b. Tachycardia c. Irregular pulse patterns d. Visual disturbances e. Decreased urinary output

ANS: B, C, D, E Heatstroke may cause an alteration in neurologic function. Other symptoms include visual disturbances, dizziness, nausea, and a weak, rapid, irregular pulse.

An intraaortic balloon pump (IABP) is being used for a patient who is in cardiogenic shock. Which assessment data indicate to the nurse that the goals of treatment with the IABP are being met? a. Urine output of 25 mL/hr b. Heart rate of 110 beats/minute c. Cardiac output (CO) of 5 L/min d. Stroke volume (SV) of 40 mL/beat

ANS: C A CO of 5 L/min is normal and indicates that the IABP has been successful in treating the shock. The low SV signifies continued cardiogenic shock. The tachycardia and low urine output also suggest continued cardiogenic shock.

8. The emergency department (ED) nurse receives report that a patient involved in a motor vehicle crash is being transported to the facility with an estimated arrival in 1 minute. In preparation for the patient's arrival, the nurse will obtain a. hypothermia blanket. b. lactated Ringer's solution. c. two 14-gauge IV catheters. d. dopamine (Intropin) infusion.

ANS: C A patient with multiple trauma may require fluid resuscitation to prevent or treat hypovolemic shock, so the nurse will anticipate the need for 2 large bore IV lines to administer normal saline. Lactated Ringer's solution should be used cautiously and will not be ordered until the patient has been assessed for possible liver abnormalities. Vasopressor infusion is not used as the initial therapy for hypovolemic shock. Patients in shock need to be kept warm not cool. DIF: Cognitive Level: Apply (application) REF: 1641 TOP: Nursing Process: Planning MSC: NCLEX: Physiological Integrity

The nurse notes thick, white secretions in the endotracheal tube (ET) of a patient who is receiving mechanical ventilation. Which intervention will be most effective in addressing this problem? a. Increase suctioning to every hour. b. Reposition the patient every 1 to 2 hours. c. Add additional water to the patient's enteral feedings. d. Instill 5 mL of sterile saline into the ET before suctioning.

ANS: C Because the patient's secretions are thick, better hydration is indicated. Suctioning every hour without any specific evidence for the need will increase the incidence of mucosal trauma and would not address the etiology of the ineffective airway clearance. Instillation of saline does not liquefy secretions and may decrease the SpO2. Repositioning the patient is appropriate but will not decrease the thickness of secretions.

14. A restaurant patron sitting at the next table begins to choke. The patron yells, "I'm choking! I can't breathe!" The first responder should: a. initiate the Heimlich maneuver immediately. b. strike the victim sharply between the scapulae. c. encourage him to keep coughing and deep breathe. d. offer him a small sip of fluid.

ANS: C Because the victim can cough and speak, the airway is not compromised. Support his efforts to clear the foreign matter by coughing. The Heimlich maneuver can be initiated at such a time that the victim's airway becomes occluded.

23. Which finding about a patient who is receiving vasopressin (Pitressin) to treat septic shock ismost important for the nurse to communicate to the health care provider? a. The patient's urine output is 18 mL/hr. b. The patient's heart rate is 110 beats/minute. c. The patient is complaining of chest pain. d. The patient's peripheral pulses are weak.

ANS: C Because vasopressin is a potent vasoconstrictor, it may decrease coronary artery perfusion. The other information is consistent with the patient's diagnosis and should be reported to the health care provider but does not indicate a need for a change in therapy. DIF: Cognitive Level: Apply (application) REF: 1643 OBJ: Special Questions: Prioritization TOP: Nursing Process: Assessment MSC: NCLEX: Physiological Integrity

1. A diving accident occurs at the community pool and the victim is conscious and in pain. Which intervention is most appropriate pending the arrival of emergency medical personnel? a. Pulled up onto the side of the pool b. Pulled to the shallow end and covered with a towel c. Left in the pool and supported by a large float d. Supported in the pool while CPR is attempted

ANS: C Care should be taken to avoid movement of the patient and increasing injury to the spinal cord. Leaving the patient in the pool and supporting the patient on a float will not increase a possible spinal injury. Pulling the patient may cause further injury to the spine. CPR is not indicated as the patient is not experiencing cardiopulmonary arrest.

15. A client in shock has been started on dopamine. What assessment finding requires the nurse to communicate with the provider immediately? a. Blood pressure of 98/68 mm Hg b. Pedal pulses 1+/4+ bilaterally c. Report of chest heaviness d. Urine output of 32 mL/hr

ANS: C Chest heaviness or pain indicates myocardial ischemia, a possible adverse effect of dopamine. While taking dopamine, the oxygen requirements of the heart are increased due to increased myocardial workload, and may cause ischemia. Without knowing the client's previous blood pressure or pedal pulses, there is not enough information to determine if these are an improvement or not. A urine output of 32 mL/hr is acceptable.

A worker in a department store fell through a plate glass window, causing a deep laceration on the right mid-thigh that is pumping bright red arterial blood. The best initial intervention of the first responder should be to: a. elevate the leg. b. bunch up the worker's shirt and press it against the wound. c. press the palm of hand in the groin to compress the femoral artery. d. tie the worker's belt tightly around his upper thigh to stop bleeding.

ANS: C Compression of the artery is the best initial move, followed by compression with dressing and elevation.

A victim of a knife fight is found lying in a parking lot with a loop of bowel protruding from an abdominal wound. The first responder should: a. attempt to replace the bowel back into the abdomen. b. wrap the victim's shirt tightly around his body. c. cover the evisceration with a plastic shopping bag. d. assist the victim to flex his thighs against his abdomen.

ANS: C Covering evisceration with a nonadhesive covering will keep the bowel moist. Attempts to return the bowel into the abdomen may result in further injury. Tightly wrapping the shirt around the body may compromise circulation. Flexion of the thighs onto the abdomen may compress and cause further damage to the bowel.

The nurse notes premature ventricular contractions (PVCs) while suctioning a patient's endotracheal tube. Which action by the nurse is a priority? a. Decrease the suction pressure to 80 mm Hg. b. Document the dysrhythmia in the patient's chart. c. Stop and ventilate the patient with 100% oxygen. d. Give antidysrhythmic medications per protocol.

ANS: C Dysrhythmias during suctioning may indicate hypoxemia or sympathetic nervous system stimulation. The nurse should stop suctioning and ventilate the patient with 100% oxygen. Lowering the suction pressure will decrease the effectiveness of suctioning without improving the hypoxemia. Because the PVCs occurred during suctioning, there is no need for antidysrhythmic medications (which may have adverse effects) unless they recur when the suctioning is stopped and patient is well oxygenated.

The nurse assesses the frostbite on the victim's hands and feet to be second-degree frostbite because the skin is: a. reddened and has hard white plaques. b. waxy and has sensory deficits. c. reddened and has blisters filled with milky fluid. d. waxy and has blisters filled with blood.

ANS: C Frostbite is categorized by degree of injury, much like burns. The appearance of a first-degree injury includes reddened skin, swelling, waxy appearance, hard white plaques, and sensory deficit. Second-degree injury also has redness and swelling and formation of blisters filled with clear or milky fluid that form within 24 hours of injury. In third-degree injury, the blisters are blood filled followed by black eschar forming over several weeks. Fourth-degree injury involves full-thickness damage affecting muscles, tendons, and bone, resulting in tissue loss.

The family members of a patient who has just been admitted to the intensive care unit (ICU) with multiple traumatic injuries have just arrived in the ICU waiting room. Which action should the nurse take next? a. Explain ICU visitation policies and encourage family visits. b. Immediately take the family members to the patient's bedside. c. Describe the patient's injuries and the care that is being provided. d. Invite the family to participate in a multidisciplinary care conference.

ANS: C Lack of information is a major source of anxiety for family members and should be addressed first. Family members should be prepared for the patient's appearance and the ICU environment before visiting the patient for the first time. ICU visiting should be individualized to each patient and family rather than being dictated by rigid visitation policies. Inviting the family to participate in a multidisciplinary conference is appropriate but should not be the initial action by the nurse.

A student nurse is assisting with the care of a 50-year-old man who is being treated in the emergency department for hypothermia. The student asks the charge nurse why the patient is having his heart monitored. The nurse's best explanation would be that the heart is being monitored because serious arrhythmias can be caused by: a. rapidly rising blood pressure from infusing IV fluids. b. adrenal output of epinephrine in response to cold stress. c. lactic acid being shunted from pooled blood in the extremities. d. vasodilation from the warming process.

ANS: C Lactic acid in the blood that was pooled in the extremities while being exposed to cold will shunt back to the heart through systemic perfusion as the warming process becomes effective. The lactic acid can cause arrhythmias.

The nurse is caring for a patient who has an intraaortic balloon pump in place. Which action should be included in the plan of care? a. Position the patient supine at all times. b. Avoid the use of anticoagulant medications. c. Measure the patient's urinary output every hour. d. Provide passive range of motion for all extremities.

ANS: C Monitoring urine output will help determine whether the patient's cardiac output has improved and also help monitor for balloon displacement. The head of the bed can be elevated up to 30 degrees. Heparin is used to prevent thrombus formation. Limited movement is allowed for the extremity with the balloon insertion site to prevent displacement of the balloon.

3. A 19-year-old patient with massive trauma and possible spinal cord injury is admitted to the emergency department (ED). Which assessment finding by the nurse will help confirm a diagnosis of neurogenic shock? a. Inspiratory crackles. b. Cool, clammy extremities. c. Apical heart rate 45 beats/min. d. Temperature 101.2° F (38.4° C).

ANS: C Neurogenic shock is characterized by hypotension and bradycardia. The other findings would be more consistent with other types of shock. DIF: Cognitive Level: Understand (comprehension) REF: 1634 TOP: Nursing Process: Assessment MSC: NCLEX: Physiological Integrity

10. A student nurse is caring for a client who will be receiving sodium nitroprusside (Nipride) via IV infusion. What action by the student causes the registered nurse to intervene? a. Assessing the IV site before giving the drug b. Obtaining a programmable ("smart") IV pump c. Removing the IV bag from the brown plastic cover d. Taking and recording a baseline set of vital signs

ANS: C Nitroprusside degrades in the presence of light, so it must be protected by leaving it in the original brown plastic bag when infusing. The other actions are correct, although a "smart" pump is not necessarily required if the facility does not have them available. The drug must be administered via an IV pump, although the programmable pump is preferred for safety.

An older patient with cardiogenic shock is cool and clammy. Hemodynamic monitoring indicates a high systemic vascular resistance (SVR). Which intervention should the nurse anticipate? a. Increase the rate for the dopamine infusion. b. Decrease the rate for the nitroglycerin infusion. c. Increase the rate for the sodium nitroprusside infusion. d. Decrease the rate for the 5% dextrose in normal saline (D5/.9 NS) infusion.

ANS: C Nitroprusside is an arterial vasodilator and will decrease the SVR and afterload, which will improve cardiac output. Changes in the D5/.9 NS and nitroglycerin infusions will not directly decrease SVR. Increasing the dopamine will tend to increase SVR.

4. An older patient with cardiogenic shock is cool and clammy and hemodynamic monitoring indicates a high systemic vascular resistance (SVR). Which intervention should the nurse anticipate doing next? a. Increase the rate for the dopamine (Intropin) infusion. b. Decrease the rate for the nitroglycerin (Tridil) infusion. c. Increase the rate for the sodium nitroprusside (Nipride) infusion. d. Decrease the rate for the 5% dextrose in normal saline (D5/.9 NS) infusion.

ANS: C Nitroprusside is an arterial vasodilator and will decrease the SVR and afterload, which will improve cardiac output. Changes in the D5/.9 NS and nitroglycerin infusions will not directly decrease SVR. Increasing the dopamine will tend to increase SVR. DIF: Cognitive Level: Apply (application) REF: 1644 TOP: Nursing Process: Planning MSC: NCLEX: Physiological Integrity

When caring for a patient with pulmonary hypertension, which parameter is most appropriate for the nurse to monitor to evaluate the effectiveness of the treatment? a. Central venous pressure (CVP) b. Systemic vascular resistance (SVR) c. Pulmonary vascular resistance (PVR) d. Pulmonary artery wedge pressure (PAWP)

ANS: C PVR is a major contributor to pulmonary hypertension, and a decrease would indicate that pulmonary hypertension was improving. The other parameters also may be monitored but do not directly assess for pulmonary hypertension.

When the nurse educator is evaluating the skills of a new registered nurse (RN) caring for patients experiencing shock, which action by the new RN indicates a need for more education? a. Placing the pulse oximeter on the ear for a patient with septic shock b. Keeping the head of the bed flat for a patient with hypovolemic shock c. Maintaining a cool room temperature for a patient with neurogenic shock d. Increasing the nitroprusside infusion rate for a patient with a very high SVR

ANS: C Patients with neurogenic shock have poikilothermia. The room temperature should be kept warm to avoid hypothermia. The other actions by the new RN are appropriate.

To evaluate the effectiveness of the pantoprazole (Protonix) ordered for a patient with systemic inflammatory response syndrome (SIRS), which assessment will the nurse perform? a. Auscultate bowel sounds. c. Check stools for occult blood. b. Ask the patient about nausea. d. Palpate for abdominal tenderness.

ANS: C Proton pump inhibitors are given to decrease the risk for stress ulcers in critically ill patients. The other assessments will also be done, but these will not help in determining the effectiveness of the pantoprazole administration.

The nurse reminds a group of CNAs that for a patient with an elevated temperature, the quickest and simplest technique to reduce the temperature is to: a. apply ice packs to the groin. b. bathe in tepid water. c. remove clothing and bed linen. d. give chilled drinks.

ANS: C Removing the patient's clothing and bed linen covering the patient is a quick, simple, and usually effective way to reduce temperature. The application of ice packs may result in excessive cooling and result in shivering, which acts to increase metabolic rate. Bathing in tepid water is effective but requires more time and interaction than simply removing clothing and bed linens. Chilled drinks will not adequately reduce the total body temperature.

The nurse cautions that, when cooling down a victim of heatstroke, one must be careful to prevent shivering because shivering can cause: a. a paralytic ileus. b. cardiac arrhythmias. c. an increase in temperature. d. a seizure.

ANS: C Shivering is a homeostatic activity that generates heat and increases body temperature.

A nurse is weaning a 68-kg male patient who has chronic obstructive pulmonary disease (COPD) from mechanical ventilation. Which patient assessment finding indicates that the weaning protocol should be stopped? a. The patient's heart rate is 97 beats/min. b. The patient's oxygen saturation is 93%. c. The patient respiratory rate is 32 breaths/min. d. The patient's spontaneous tidal volume is 450 mL.

ANS: C Tachypnea is a sign that the patient's work of breathing is too high to allow weaning to proceed. The patient's heart rate is within normal limits, although the nurse should continue to monitor it. An oxygen saturation of 93% is acceptable for a patient with COPD. A spontaneous tidal volume of 450 mL is within the acceptable range.

13. An 80-year-old woman is brought to the emergency department by her daughter, who found the woman unconscious in her garage sitting in her car. A significant assessment of this patient as to cause of her condition would be: a. temperature, 97.6° F; pulse, 98; and blood pressure, 110/60. b. O2 saturation of 78%. c. cherry red mucous membranes. d. cold extremities.

ANS: C The cherry red mucous membranes are classic signs of carbon monoxide poisoning; unfortunately, they are very late signs.

A patient who has neurogenic shock is receiving a phenylephrine infusion through a right forearm IV. Which assessment finding obtained by the nurse indicates a need for immediate action? a. The patient's heart rate is 58 beats/min. b. The patient's extremities are warm and dry. c. The patient's IV infusion site is cool and pale. d. The patient's urine output is 28 mL over the past hour.

ANS: C The coldness and pallor at the infusion site suggest extravasation of the phenylephrine. The nurse should discontinue the IV and, if possible, infuse the drug into a central line. An apical pulse of 58 beats/min is typical for neurogenic shock but does not indicate an immediate need for nursing intervention. A 28-mL urinary output over 1 hour would require the nurse to monitor the output over the next hour, but an immediate change in therapy is not indicated. Warm, dry skin is consistent with early neurogenic shock, but it does not indicate a need for a change in therapy or immediate action.

21. The patient with neurogenic shock is receiving a phenylephrine (Neo-Synephrine) infusion through a right forearm IV. Which assessment finding obtained by the nurse indicates a need forimmediate action? a. The patient's heart rate is 58 beats/minute. b. The patient's extremities are warm and dry. c. The patient's IV infusion site is cool and pale. d. The patient's urine output is 28 mL over the last hour.

ANS: C The coldness and pallor at the infusion site suggest extravasation of the phenylephrine. The nurse should discontinue the IV and, if possible, infuse the medication into a central line. An apical pulse of 58 is typical for neurogenic shock but does not indicate an immediate need for nursing intervention. A 28-mL urinary output over 1 hour would require the nurse to monitor the output over the next hour, but an immediate change in therapy is not indicated. Warm, dry skin is consistent with early neurogenic shock, but it does not indicate a need for a change in therapy or immediate action. DIF: Cognitive Level: Apply (application) REF: 1643 OBJ: Special Questions: Prioritization TOP: Nursing Process: Assessment MSC: NCLEX: Physiological Integrity

20. A patient who has been involved in a motor vehicle crash arrives in the emergency department (ED) with cool, clammy skin; tachycardia; and hypotension. Which intervention ordered by the health care provider should the nurse implement first? a. Insert two large-bore IV catheters. b. Initiate continuous electrocardiogram (ECG) monitoring. c. Provide oxygen at 100% per non-rebreather mask. d. Draw blood to type and crossmatch for transfusions.

ANS: C The first priority in the initial management of shock is maintenance of the airway and ventilation. ECG monitoring, insertion of IV catheters, and obtaining blood for transfusions should also be rapidly accomplished but only after actions to maximize oxygen delivery have been implemented. DIF: Cognitive Level: Apply (application) REF: 1641 OBJ: Special Questions: Prioritization TOP: Nursing Process: Implementation MSC: NCLEX: Physiological Integrity

Which assessment finding obtained by the nurse when caring for a patient receiving mechanical ventilation indicates the need for suctioning? a. The patient's oxygen saturation is 93%. b. The patient was last suctioned 6 hours ago. c. The patient's respiratory rate is 32 breaths/minute. d. The patient has occasional audible expiratory wheezes.

ANS: C The increase in respiratory rate indicates that the patient may have decreased airway clearance and requires suctioning. Suctioning is done when patient assessment data indicate that it is needed, not on a scheduled basis. Occasional expiratory wheezes do not indicate poor airway clearance, and suctioning the patient may induce bronchospasm and increase wheezing. An oxygen saturation of 93% is acceptable and does not suggest that immediate suctioning is needed.

A patient is admitted to the emergency department (ED) for shock of unknown etiology. The first action by the nurse should be to a. obtain the blood pressure. b. check the level of orientation. c. administer supplemental oxygen. d. obtain a 12-lead electrocardiogram.

ANS: C The initial actions of the nurse are focused on the ABCs—airway, breathing, and circulation—and administration of O2 should be done first. The other actions should be accomplished as rapidly as possible after providing O2.

After reviewing the information shown in the accompanying figure for a patient with pneumonia and sepsis, which information is most important to report to the health care provider? Physical Assessment: Petechiae noted on chest & legs; Crackles heard bilaterally in lung bases; no redness or swelling at central line IV site Lab Data: BUN 34 mg, Hct 30, Plt 50,000 VS: Temp 100; Pulse 102; RR 26; BP 110/60, O2 93 via NC a. Temperature and IV site appearance b. Oxygen saturation and breath sounds c. Platelet count and presence of petechiae d. Blood pressure, pulse rate, respiratory rate.

ANS: C The low platelet count and presence of petechiae suggest that the patient may have disseminated intravascular coagulation and that multiple organ dysfunction syndrome is developing. The other information will also be discussed with the health care provider but does not indicate that the patient's condition is deteriorating or that a change in therapy is needed immediately.

25. After reviewing the information shown in the accompanying figure for a patient with pneumonia and sepsis, which information is most important to report to the health care provider? a. Temperature and IV site appearance b. Oxygen saturation and breath sounds c. Platelet count and presence of petechiae d. Blood pressure, pulse rate, respiratory rate.

ANS: C The low platelet count and presence of petechiae suggest that the patient may have disseminated intravascular coagulation and that multiple organ dysfunction syndrome (MODS) is developing. The other information will also be discussed with the health care provider but does not indicate that the patient's condition is deteriorating or that a change in therapy is needed immediately. DIF: Cognitive Level: Analyze (analysis) REF: 1640 OBJ: Special Questions: Prioritization TOP: Nursing Process: Assessment MSC: NCLEX: Physiological Integrity

Which action is a priority for the nurse to take when the low pressure alarm sounds for a patient who has an arterial line in the left radial artery? a. Fast flush the arterial line. b. Check the left hand for pallor. c. Assess for cardiac dysrhythmias. d. Rezero the monitoring equipment.

ANS: C The low pressure alarm indicates a drop in the patient's blood pressure, which may be caused by cardiac dysrhythmias. There is no indication to rezero the equipment. Pallor of the left hand would be caused by occlusion of the radial artery by the arterial catheter, not by low pressure. There is no indication of a need for flushing the line.

The nurse notes that a patient's endotracheal tube (ET), which was at the 22-cm mark, is now at the 25-cm mark and the patient is anxious and restless. Which action should the nurse take next? a. Offer reassurance to the patient. b. Bag the patient at an FIO2 of 100%. c. Listen to the patient's breath sounds. d. Notify the patient's health care provider.

ANS: C The nurse should first determine whether the ET tube has been displaced into the right mainstem bronchus by listening for unilateral breath sounds. If so, assistance will be needed to reposition the tube immediately. The other actions are also appropriate, but detection and correction of tube malposition are the most critical actions.

When the patient complains, "If this viral infection I have right now can't be helped by antibiotics, why am I taking this expensive acyclovir?" How should the nurse respond? a."Acyclovir is an antiviral drug that kills viruses." b."Acyclovir is given to many patients with viral infections." c."Acyclovir is an antiviral drug that prevents your infection from becoming worse." d."Acyclovir helps strengthen your immune system."

ANS: C The patient currently has a viral infection; acyclovir is an antiviral drug that will decrease the virulence of the infection if started in the early phase of the infection. The drug may not kill the virus and is not given frequently to patients with viruses. Acyclovir will not strengthen the immune system. PTS: 1 DIF: Cognitive Level: Application REF: 101 OBJ: 8 (theory) TOP: Viruses KEY: Nursing Process Step: Implementation MSC: NCLEX: Safe, Effective Care Environment: Safety and Infection Control

A drowning victim is brought to shore and is semiconscious and breathing. The most appropriate initial action of the camp counselor will require that the victim will be placed in what position? a. Supine to receive CPR b. Supine with knees flexed c. On the side in recovery position d. Prone with head turned to side

ANS: C The patient who is breathing should be placed in the recovery position to allow the patient to vomit out water without danger of aspiration. CPR is not indicated as the patient is not experiencing the absence of cardiopulmonary activity. Lying supine or prone will not prevent aspiration in the event of vomiting.

6. A nurse caring for a client notes the following assessments: white blood cell count 3800/mm3, blood glucose level 198 mg/dL, and temperature 96.2° F (35.6° C). What action by the nurse takes priority? a. Document the findings in the client's chart. b. Give the client warmed blankets for comfort. c. Notify the health care provider immediately. d. Prepare to administer insulin per sliding scale.

ANS: C This client has several indicators of sepsis with systemic inflammatory response. The nurse should notify the health care provider immediately. Documentation needs to be thorough but does not take priority. The client may appreciate warm blankets, but comfort measures do not take priority. The client may or may not need insulin.

The nurse is caring for a patient receiving a continuous norepinephrine (Levophed) IV infusion. Which patient assessment finding indicates that the infusion rate may need to be adjusted? a. Heart rate is 58 beats/minute. b. Mean arterial pressure (MAP) is 56 mm Hg. c. Systemic vascular resistance (SVR) is elevated. d. Pulmonary artery wedge pressure (PAWP) is low.

ANS: C Vasoconstrictors such as norepinephrine (Levophed) will increase SVR, and this will increase the work of the heart and decrease peripheral perfusion. The infusion rate may need to be decreased. Bradycardia, hypotension (MAP of 56 mm Hg), and low PAWP are not associated with norepinephrine infusion.

A nurse is assessing a patient who is receiving a nitroprusside infusion to treat cardiogenic shock. Which finding indicates that the drug is effective? a. No new heart murmurs c. Warm, pink, and dry skin b. Decreased troponin level d. Blood pressure of 92/40 mm Hg

ANS: C Warm, pink, and dry skin indicates that perfusion to tissues is improved. Because nitroprusside is a vasodilator, the blood pressure may be low even if the drug is effective. Absence of a heart murmur and a decrease in troponin level are not indicators of improvement in shock.

12. A nurse is assessing a patient who is receiving a nitroprusside (Nipride) infusion to treat cardiogenic shock. Which finding indicates that the medication is effective? a. No new heart murmurs b. Decreased troponin level c. Warm, pink, and dry skin d. Blood pressure 92/40 mm Hg

ANS: C Warm, pink, and dry skin indicates that perfusion to tissues is improved. Since nitroprusside is a vasodilator, the blood pressure may be low even if the medication is effective. Absence of a heart murmur and a decrease in troponin level are not indicators of improvement in shock. DIF: Cognitive Level: Apply (application) REF: 1644 TOP: Nursing Process: Evaluation MSC: NCLEX: Physiological Integrity

After receiving 2 L of normal saline, the central venous pressure for a patient who has septic shock is 10 mm Hg, but the blood pressure is still 82/40 mm Hg. The nurse will anticipate an order for a. furosemide . c. norepinephrine . b. nitroglycerin . d. sodium nitroprusside .

ANS: C When fluid resuscitation is unsuccessful, vasopressor drugs are given to increase the systemic vascular resistance (SVR) and blood pressure and improve tissue perfusion. Furosemide would cause diuresis and further decrease the BP. Nitroglycerin would decrease the preload and further drop cardiac output and BP. Nitroprusside is an arterial vasodilator and would further decrease SVR.

The nurse is caring for a conscious patient who has symptoms consistent with hypoglycemia. After a serum glucose reading supports this diagnosis, which substance is preferred to initially increase the patient's glucose level? a. Carbonated cola beverage b. A teaspoon of white sugar c. A glass of milk d. Intravenous glucose

ANS: C When the patient is conscious, an oral glucose-containing substance is suggested. A glass of milk, glucose tablets, or hard candy is preferred.

The child with Down syndrome should be evaluated for which condition before participating in some sports? a. Hyperflexibility b. Cutis marmorata c. Atlantoaxial instability d. Speckling of iris (Brushfield spots)

ANS: C Feedback A Although hyperflexibility is characteristic of Down syndrome, it does not affect the child's ability to participate in sports. B Although cutis marmorata is characteristic of Down syndrome, it does not affect the child's ability to participate in sports. C Children with Down syndrome are at risk for atlantoaxial instability. Before participating in sports that put stress on the head and neck, a radiologic examination should be done. D Although Brushfield spots are characteristic of Down syndrome, they do not affect the child's ability to participate in sports.

Which statement best describes Fragile X syndrome? a. Chromosomal defect affecting only females b. Chromosomal defect that follows the pattern of X-linked recessive disorders c. Second most common genetic cause of cognitive impairment d. Most common cause of noninherited cognitive impairment

ANS: C Feedback A Fragile X primarily affects males. B Fragile X follows the pattern of X-linked dominant with reduced manifestation of the syndrome in female and moderate to severe dysfunction in males. C Fragile X syndrome is the most common inherited cause of cognitive impairment and the second most common cause of cognitive impairment after Down syndrome. D Fragile X is inherited.

An appropriate nursing diagnosis for a child with a cognitive dysfunction who has a limited ability to anticipate danger is a. Impaired social interaction b. Deficient knowledge c. Risk for injury d. Ineffective coping

ANS: C Feedback A Impaired social interaction is indeed a concern for the child with a cognitive disorder but does not address the limited ability to anticipate danger. B Because of the child's cognitive deficit, knowledge will not be retained and will not decrease the risk for injury. C The nurse needs to know that limited cognitive abilities to anticipate danger lead to risk for injury. D Ineffective individual coping does not address the limited ability to anticipate danger.

Throughout their life span, cognitively impaired children are less capable of managing environmental challenges and are at risk for a. Nutritional deficits b. Visual impairments c. Physical injuries d. Psychiatric problems

ANS: C Feedback A Nutritional deficits are related more to dietary habits and the caregivers' understanding of nutrition. B Visual impairments are unrelated to cognitive impairment. C Safety is a challenge for cognitively impaired children. Decreased capability to manage environmental challenges may lead to physical injuries. D Psychiatric problems may coexist with cognitive impairment; however, they are not environmental challenges.

What should the nurse keep in mind when planning to communicate with a child who has autism? a. The child has normal verbal communication. b. Expect the child to use sign language. c. The child may exhibit monotone speech and echolalia. d. The child is not listening if she is not looking at the nurse.

ANS: C Feedback A The child has impaired verbal communication and abnormalities in the production of speech. B Some autistic children may use sign language, but it is not assumed. C Children with autism have abnormalities in the production of speech such as a monotone voice or echolalia, or inappropriate volume, pitch, rate, rhythm, or intonation. D Children with autism often are reluctant to initiate direct eye contact.

What action is contraindicated when a child with Down syndrome is hospitalized? a. Determine the child's vocabulary for specific body functions. b. Assess the child's hearing and visual capabilities. c. Encourage parents to leave the child alone for extended periods of time. d. Have meals served at the child's usual meal times.

ANS: C Feedback A To communicate effectively with the child, it is important to know the child's particular vocabulary for specific body functions. B Children with Down syndrome have a high incidence of hearing loss and vision problems and should have hearing and vision assessed whenever they are in a health care facility. C The child with Down syndrome needs routine schedules and consistency. Having familiar people present, especially parents, helps to decrease the child's anxiety. D Routine schedules and consistency are important to children.

The emergency department (ED) nurse receives report that a seriously injured patient involved in a motor vehicle crash is being transported to the facility with an estimated arrival in 5 minutes. In preparation for the patient's arrival, the nurse will obtain a. a dopamine infusion. c. lactated Ringer's solution. b. a hypothermia blanket. d. two 16-gauge IV catheters.

ANS: D A patient with multiple trauma may require fluid resuscitation to prevent or treat hypovolemic shock, so the nurse will anticipate the need for 2 large-bore IV lines to administer normal saline. Lactated Ringer's solution should be used cautiously and will not be ordered until the patient has been assessed for possible liver abnormalities. Vasopressor infusion is not used as the initial therapy for hypovolemic shock. Patients in shock need to be kept warm not cool.

The nurse is providing infection control teaching to a patient. Which patient statement warrants additional patient teaching? a."It is important that I get my whooping cough vaccination as directed by my health care provider." b."Getting plenty of sleep each night will help my immune system." c."I should wash my hands before preparing my food." d."It is important that I take my antibiotic until my symptoms have completely resolved."

ANS: D Antibiotics must be completed in entirety. Partial completion of a protocol of prescribed antimicrobial medication can cause a pathogen to become resistant to that particular drug. Vaccinations, adequate rest, and proper hand hygiene are important infection control measures. PTS: 1 DIF: Cognitive Level: Application REF: 101, 113, Table 6-7 OBJ: 1 (theory) TOP: Infection Control Measures KEY: Nursing Process Step: Evaluation MSC: NCLEX: Safe, Effective Care Environment: Safety and Infection Control

Which assessment information is most important for the nurse to obtain when evaluating whether treatment of a patient with anaphylactic shock has been effective? a. Heart rate c. Blood pressure b. Orientation d. Oxygen saturation

ANS: D Because the airway edema that is associated with anaphylaxis can affect airway and breathing, the O2 saturation is the most critical assessment. Improvements in the other assessments will also be expected with effective treatment of anaphylactic shock.

The nurse is reviewing the physician's notes on a patient's chart. The nurse notes that the patient demonstrated Cullen's sign. The nurse correctly recognizes that this patient most likely had which manifestation? a. Sharp flank pain b. Pain in the upper right quadrant of the abdomen c. Pain with inspiration d. A bluish tinge around the umbilicus

ANS: D Cullen's sign refers to a bluish tinge around the umbilicus. It may be noted in the presence of internal abdominal hemorrhage.

Which action will the nurse need to do when preparing to assist with the insertion of a pulmonary artery catheter? a. Determine if the cardiac troponin level is elevated. b. Auscultate heart and breath sounds during insertion. c. Place the patient on NPO status before the procedure. d. Attach cardiac monitoring leads before the procedure.

ANS: D Dysrhythmias can occur as the catheter is floated through the right atrium and ventricle, and it is important for the nurse to monitor for these during insertion. Pulmonary artery catheter insertion does not require anesthesia, and the patient will not need to be NPO. Changes in cardiac troponin or heart and breath sounds are not expected during pulmonary artery catheter insertion.

To verify the correct placement of an oral endotracheal tube (ET) after insertion, the best initial action by the nurse is to a. auscultate for the presence of bilateral breath sounds. b. obtain a portable chest x-ray to check tube placement. c. observe the chest for symmetric chest movement with ventilation. d. use an end-tidal CO2 monitor to check for placement in the trachea.

ANS: D End-tidal CO2 monitors are currently recommended for rapid verification of ET placement. Auscultation for bilateral breath sounds and checking chest expansion are also used, but they are not as accurate as end-tidal CO2 monitoring. A chest x-ray confirms the placement but is done after the tube is secured.

The nurse who suffers an accidental needle stick following administration of an intramuscular injection to a patient anticipates that facility protocol will suggest immediate treatment with which type of immunotherapy? a.IgM b.IgD c.Ig A d.IgG

ANS: D IgG is frequently given to provide passive immunity until the body's own immune system can defend itself; therefore, it would most likely be a component of a health care facility's initial treatment protocol for accidental needle sticks. IgM, IgD, and IgA would not be indicated. PTS: 1 DIF: Cognitive Level: Comprehension REF: 104, Table 6-2 OBJ: 1 (theory) TOP: Immunoglobulins KEY: Nursing Process Step: Planning MSC: NCLEX: Health Promotion and Maintenance

The nurse uses a visual aid to demonstrate how which antibody attaches to the antigen to clear the pathogen from the body? a.IgA b.IgD c.IgG d.IgM

ANS: D Immunoglobulin M (IgM) is the antibody that recognizes the foreign protein and attaches itself to it in order to clear the pathogen from the body. PTS: 1 DIF: Cognitive Level: Knowledge REF: 104, Table 6-2 OBJ: 2 (theory) TOP: Antibodies KEY: Nursing Process Step: Implementation MSC: NCLEX: Physiological Integrity: Physiological Adaptation

The nurse is caring for a patient with a subarachnoid hemorrhage who is intubated and placed on a mechanical ventilator with 10 cm H2O of peak end-expiratory pressure (PEEP). When monitoring the patient, the nurse will need to notify the health care provider immediately if the patient develops a. oxygen saturation of 93%. b. respirations of 20 breaths/minute. c. green nasogastric tube drainage. d. increased jugular venous distention.

ANS: D Increases in jugular venous distention in a patient with a subarachnoid hemorrhage may indicate an increase in intracranial pressure (ICP) and that the PEEP setting is too high for this patient. A respiratory rate of 20, O2 saturation of 93%, and green nasogastric tube drainage are within normal limits.

To which entity should the home health nurse make a referral in order to supply a home-bound older adult with a daily meal? a.A community food bank b.The Salvation Army c.An agency supplying food stamps d.Meals on Wheels

ANS: D Meals on Wheels provides a large, nutritious meal to home-bound people. A community food bank, the Salvation Army, and food stamps would not adequately assist a home-bound individual. PTS: 1 DIF: Cognitive Level: Comprehension REF: 103, Older Adult Care Points OBJ: 1 (clinical) TOP: Nutrition KEY: Nursing Process Step: Implementation MSC: NCLEX: Health Promotion and Maintenance

The nurse is caring for a patient with C. difficile infection. Which action is most important for the nurse to take? a.Only use alcohol-based hand cleanser for hand hygiene. b.Always wear an impervious mask. c.Don proper eye protection before providing care. d.Notify housekeeping to use appropriate cleaning agents.

ANS: D Notification of housekeeping to use alcohol-free cleaners is necessary in order to eradicate the pathogen. Soap and water must be used after contact with this organism because alcohol-based hand sanitizers do not adequately kill the microorganism. A mask and eye protection are not necessary. PTS: 1 DIF: Cognitive Level: Application REF: 113 OBJ: 9 (clinical) TOP: Prevention of Health Care-Associated Infections KEY: Nursing Process Step: Implementation MSC: NCLEX: Safe, Effective Care Environment: Safety and Infection Control

When monitoring for the effectiveness of treatment for a patient with a large anterior wall myocardial infarction, the most important information for the nurse to obtain is a. central venous pressure (CVP). b. systemic vascular resistance (SVR). c. pulmonary vascular resistance (PVR). d. pulmonary artery wedge pressure (PAWP).

ANS: D PAWP reflects left ventricular end diastolic pressure (or left ventricular preload) and is a sensitive indicator of cardiac function. Because the patient is high risk for left ventricular failure, the PAWP must be monitored. An increase will indicate left ventricular failure. The other values would also provide useful information, but the most definitive measurement of changes in cardiac function is the PAWP.

16. When the nurse educator is evaluating the skills of a new registered nurse (RN) caring for patients experiencing shock, which action by the new RN indicates a need for more education? a. Placing the pulse oximeter on the ear for a patient with septic shock b. Keeping the head of the bed flat for a patient with hypovolemic shock c. Increasing the nitroprusside (Nipride) infusion rate for a patient with a high SVR d. Maintaining the room temperature at 66° to 68° F for a patient with neurogenic shock

ANS: D Patients with neurogenic shock may have poikilothermia. The room temperature should be kept warm to avoid hypothermia. The other actions by the new RN are appropriate. DIF: Cognitive Level: Apply (application) REF: 1634 | 1636 OBJ: Special Questions: Delegation TOP: Nursing Process: Evaluation MSC: NCLEX: Safe and Effective Care Environment

The nurse explains that exposure to a pathogen stimulates the macrophages to migrate to the area of infection to ingest and destroy the pathogen. This statement describes which process? a.Pathogen neutralization b.Immune response c.Antibody action d.Phagocytosis

ANS: D Phagocytosis is the process of the ingestion of a pathogen by macrophages. PTS: 1 DIF: Cognitive Level: Knowledge REF: 104 OBJ: 2 (theory) TOP: Phagocytosis KEY: Nursing Process Step: Implementation MSC: NCLEX: Physiological Integrity: Physiological Adaptation

6. To evaluate the effectiveness of the pantoprazole (Protonix) ordered for a patient with systemic inflammatory response syndrome (SIRS), which assessment will the nurse perform? a. Auscultate bowel sounds. b. Palpate for abdominal pain. c. Ask the patient about nausea. d. Check stools for occult blood.

ANS: D Proton pump inhibitors are given to decrease the risk for stress ulcers in critically ill patients. The other assessments also will be done, but these will not help in determining the effectiveness of the pantoprazole administration. DIF: Cognitive Level: Apply (application) REF: 1646 TOP: Nursing Process: Evaluation MSC: NCLEX: Physiological Integrity

The clinic nurse offers suggestions to a patient who is planning a trip to Mexico that will help prevent a protozoan infection. Which suggestion is most helpful? a."Ask the doctor for a prophylactic prescription for an antiviral drug." b."Broad-spectrum antibiotics will be most helpful if you contract a protozoan infection." c."Be sure to practice good hand hygiene while on your vacation." d."It would be best if you drank bottled water while on your trip."

ANS: D Protozoa frequently live in the water and soil and cause infection by ingestion of the parasite. Water in many foreign countries contains protozoa, so drinking bottled water is the best suggestion. PTS: 1 DIF: Cognitive Level: Application REF: 101 OBJ: 1 (theory) TOP: Protozoa KEY: Nursing Process Step: Implementation MSC: NCLEX: Physiological Integrity: Reduction of Risk Potential

A spectator at the Little League playoffs in August in Texas faints in the sun-drenched stands. His face is flushed and his skin is hot to the touch. The first responder should assist him to: a. lie down on the bleacher seat. b. drink a large iced drink. c. be seated in the stands, shielded from the sun with an umbrella. d. a shady area, and sprinkle his clothing with water.

ANS: D Remove the victim from the sun and cool by evaporation until emergency medical personnel arrive.

The nurse using the memory prompt for emergency care, ABCDE, is aware that the "E" stands for: a. end. b. execute. c. expedite. d. expose.

ANS: D The "E" stands for expose. This reminder is to assist the first responder to assess for other injuries that may be hidden under clothing.

After change-of-shift report on a ventilator weaning unit, which patient should the nurse assess first? a. Patient who failed a spontaneous breathing trial and has been placed in a rest mode on the ventilator b. Patient who is intubated and has continuous partial pressure end-tidal CO2 (PETCO2) monitoring c. Patient with a central venous oxygen saturation (ScvO2) of 69% while on bilevel positive airway pressure (BiPAP) d. Patient who was successfully weaned and extubated 4 hours ago and now has no urine output for the last 6 hours

ANS: D The decreased urine output may indicate acute kidney injury or that the patient's cardiac output and perfusion of vital organs have decreased. Any of these causes would require rapid action. The data about the other patients indicate that their conditions are stable and do not require immediate assessment or changes in their care. Continuous PETCO2 monitoring is frequently used when patients are intubated. The rest mode should be used to allow patient recovery after a failed SBT, and an ScvO2 of 69% is within normal limits.

After change-of-shift report, which patient should the progressive care nurse assess first? a. Patient who was extubated in the morning and has a temperature of 101.4° F (38.6° C) b. Patient with bilevel positive airway pressure (BiPAP) for sleep apnea whose respiratory rate is 16 c. Patient with arterial pressure monitoring who is 2 hours post-percutaneous coronary intervention who needs to void d. Patient who is receiving IV heparin for a venous thromboembolism and has a partial thromboplastin time (PTT) of 98 sec

ANS: D The findings for this patient indicate high risk for bleeding from an elevated (nontherapeutic) PTT. The nurse needs to adjust the rate of the infusion (dose) per the health care provider's parameters. The patient with BiPAP for sleep apnea has a normal respiratory rate. The patient recovering from the percutaneous coronary intervention will need to be assisted with voiding and this task could be delegated to unlicensed assistive personnel. The patient with a fever may be developing ventilator-associated pneumonia, but addressing the bleeding risk is a higher priority.

The nurse educator is evaluating the care that a new registered nurse (RN) provides to a patient receiving mechanical ventilation. Which action by the new RN indicates the need for more education? a. The RN increases the FIO2 to 100% before suctioning. b. The RN secures a bite block in place using adhesive tape. c. The RN asks for assistance to reposition the endotracheal tube. d. The RN positions the patient with the head of bed at 10 degrees.

ANS: D The head of the patient's bed should be positioned at 30 to 45 degrees to prevent ventilator-associated pneumonia. The other actions by the new RN are appropriate.

While waiting for cardiac transplantation, a patient with severe cardiomyopathy has a ventricular assist device (VAD) implanted. When planning care for this patient, the nurse should anticipate a. giving immunosuppressive medications. b. preparing the patient for a permanent VAD. c. teaching the patient the reason for complete bed rest. d. monitoring the surgical incision for signs of infection.

ANS: D The insertion site for the VAD provides a source for transmission of infection to the circulatory system and requires frequent monitoring. Patient's with VADs are able to have some mobility and may not be on bed rest. The VAD is a bridge to transplantation, not a permanent device. Immunosuppression is not necessary for nonbiologic devices like the VAD.

To maintain proper cuff pressure of an endotracheal tube (ET) when the patient is on mechanical ventilation, the nurse should a. inflate the cuff with a minimum of 10 mL of air. b. inflate the cuff until the pilot balloon is firm on palpation. c. inject air into the cuff until a manometer shows 15 mm Hg pressure. d. inject air into the cuff until a slight leak is heard only at peak inflation.

ANS: D The minimal occluding volume technique involves injecting air into the cuff until an air leak is present only at peak inflation. The volume to inflate the cuff varies with the ET and the patient's size. Cuff pressure should be maintained at 20 to 25 mm Hg. An accurate assessment of cuff pressure cannot be obtained by palpating the pilot balloon.

The nurse responds to a ventilator alarm and finds the patient lying in bed holding the endotracheal tube (ET). Which action should the nurse take next? a. Activate the rapid response team. b. Provide reassurance to the patient. c. Call the health care provider to reinsert the tube. d. Manually ventilate the patient with 100% oxygen.

ANS: D The nurse should ensure maximal patient oxygenation by manually ventilating with a bag-valve-mask system. Offering reassurance to the patient, notifying the health care provider about the need to reinsert the tube, and activating the rapid response team are also appropriate after the nurse has stabilized the patient's oxygenation.

Four hours after mechanical ventilation is initiated for a patient with chronic obstructive pulmonary disease (COPD), the patient's arterial blood gas (ABG) results include a pH of 7.51, PaO2 of 82 mm Hg, PaCO2 of 26 mm Hg, and HCO3- of 23 mEq/L (23 mmol/L). The nurse will anticipate the need to a. increase the FIO2. b. increase the tidal volume. c. increase the respiratory rate. d. decrease the respiratory rate.

ANS: D The patient's PaCO2 and pH indicate respiratory alkalosis caused by too high a respiratory rate. The PaO2 is appropriate for a patient with COPD and increasing the respiratory rate and tidal volume would further lower the PaCO2.

When assisting with the placement of a pulmonary artery (PA) catheter, the nurse notes that the catheter is correctly placed when the monitor shows a a. typical PA pressure waveform. b. tracing of the systemic arterial pressure. c. tracing of the systemic vascular resistance. d. typical PA wedge pressure (PAWP) tracing.

ANS: D The purpose of a PA line is to measure PAWP, so the catheter is floated through the pulmonary artery until the dilated balloon wedges in a distal branch of the pulmonary artery, and the PAWP readings are available. After insertion, the balloon is deflated and the PA waveform will be observed. Systemic arterial pressures are obtained using an arterial line and the systemic vascular resistance is a calculated value, not a waveform.

When caring for the patient with a pulmonary artery (PA) pressure catheter, the nurse observes that the PA waveform indicates that the catheter is in the wedged position. Which action should the nurse take next? a. Zero balance the transducer. b. Activate the fast flush system. c. Notify the health care provider. d. Deflate and reinflate the PA balloon.

ANS: D When the catheter is in the wedge position, blood flow past the catheter is obstructed, placing the patient at risk for pulmonary infarction. A health care provider or advanced practice nurse should be called to reposition the catheter. The other actions will not correct the wedging of the PA catheter.

The best setting for daytime care for a 5-year-old autistic child whose mother works is a. Private day care b. Public school c. His own home with a sitter d. A specialized program that facilitates interaction by use of behavioral methods

ANS: D Feedback A Daycare programs generally do not have resources to meet the needs of severely impaired children. B To best meet the needs of an autistic child, the public school may refer the child to a specialized program. C A sitter might not have the skills to interact with an autistic child. D Autistic children can benefit from specialized educational programs that address their special needs.

The nurse is providing counseling to the mother of a child diagnosed with fragile X syndrome. She explains to the mother that fragile X syndrome is a. Most commonly seen in girls b. Acquired after birth c. Usually transmitted by the male carrier d. Usually transmitted by the female carrier

ANS: D Feedback A Fragile X syndrome is most common in males. B Fragile X syndrome is congenital. C Fragile X syndrome is not transmitted by a male carrier. D The gene causing fragile X syndrome is transmitted by the mother.

A neighbor is found slumped over the lawn mower and is unconscious. Arrange the interventions made by the first responder in appropriate order. a. Tell neighbor's wife to call 911. b. Assess for heartbeat. c. Initiate cardiopulmonary resuscitation (CPR) if no respiration or circulation can be assessed. d. Assess for signs of breathing. e. Shake patient and call name to assess for level of consciousness (LOC).

Ans: A E D B C

During the physical assessment of a recently born neonate, the nurse palpates the infant's femoral pulses. For which cardiac defect is the nurse looking? A.) Atrial septal defect B.) Coarctation of the aorta C.) Patent ductus arteriosus D.)Ventricular septal defect

B --Coarctation of the aorta results in diminished or absent femoral pulses. An atrial septal defect has no effect on the volume of peripheral circulation. (Minimal shunting occurs in the newborn period.) A patent ductus arteriosus has minimal effect on the volume of peripheral circulation (left-to-right shunt). A ventricular septal defect has minimal effect on the volume of peripheral circulation (left-to-right shunt)

client and her 2-week-old newborn are scheduled to be seen by the visiting nurse at home. When the nurse arrives, the client appears exhausted and the infant is crying. After comforting the client, what is the nurse's most appropriate comment? A.) "Are you feeling all right?" B.) "Tell me about your daily routine." C.) "You must be having a terrible day." D.) "When was the baby's last feeding?"

B ---Learning about the client's routine permits collection of more data. Asking whether the client is feeling all right implies that the client is not well enough to care for her baby. A negative comment closes communication. Asking about the last feeding may make the mother feel guilty about not meeting her baby's needs.

A neonate is tested for phenylketonuria (PKU) after formula feedings are initiated. The nurse explains to the parents that this is done to prevent: A.) Failure to thrive B.) Cognitive impairment C.) Growth restriction D.) Specific food allergies

B ---Screening for PKU facilitates early diagnosis and treatment, which can prevent mental retardation. Although children with untreated PKU do have problems with physical growth and may exhibit failure to thrive, the major purpose of the test is to prevent the development of cognitive impairment. Telling the parent that this test is performed to prevent specific food allergies is not accurate because this is not a test for food allergies; it tests for an inborn error of metabolism, PKU.

A client is rooming in with her newborn. The nurse sees the infant lying quietly in the bassinet with the eyes open wide. What action should the nurse take in response to the infant's behavior? A.) Brightening the lights in the room B.) Encouraging the mother to talk to her baby C.) Wrapping and then turning the infant to the side D.) Beginning physical and behavioral assessments

B --A quiet, alert state is an optimal time for infant stimulation. Bright lights are disturbing to newborns and may impede mother-infant interaction. Wrapping and then turning the infant to the side is done for the sleeping infant. Physical and behavioral assessments are not the priorities; they may be delayed.

A mother who is formula feeding her 1-month-old infant asks the nurse whether any vitamin or mineral supplements are absent from the ready-to-use formulas. The nurse responds that the supplement that is not in ready-to-use formulas is: A.) Iron B.) Fluoride C.) Vitamin K D.) Vitamin B12

B --The use of fluoride supplements has become a controversial issue. The Centers for Disease Control and Prevention still recommends it. The recommended amount is 0.25 mg daily if fluoridated water is not available. Commercial formulas are fortified with iron. The supply of vitamin K is adequate after the first week of life. Vitamin B12 supplementation is unnecessary; the formula contains vitamin B12

A nurse receives report from the laboratory on a client who was admitted for fever. The laboratory technician states that the client has "a shift to the left" on the white blood cell count. What action by the nurse is most important? a. Document findings and continue monitoring. b. Notify the provider and request antibiotics. c. Place the client in protective isolation. d. Tell the client this signifies inflammation.

B A shift to the left indicates an increase in immature neutrophils and is often seen in infections, especially those caused by bacteria. The nurse should notify the provider and request antibiotics. Documentation and teaching need to be done, but the nurse needs to do more. The client does not need protective isolation.

A client has been placed on Contact Precautions. The client's family is very afraid to visit for fear of being "contaminated" by the client. What action by the nurse is best? a. Explain to them that these precautions are mandated by law. b. Inform them that the infection is the issue, not the client. c. Reassure the family that they will not get the infection. d. Tell the family it is important that they visit the client.

B Families and clients often have negative reactions to isolation precautions. The nurse can explain that the infection is the problem, not the client, and encourage them to visit because following the precautions will prevent them from acquiring the infection. The other options do not give the family useful information to help them make an informed decision.

A client with an infection has a fever. What actions by the nurse help increase the client's comfort? (Select all that apply.) a. Administer antipyretics around the clock. b. Change the client's gown and linens when damp. c. Offer cool fluids to the client frequently. d. Place ice bags in the armpits and groin. e. Provide a fan to help cool the client.

B, C Comfort measures appropriate for this client include offering frequent cool drinks, and changing linens or the gown when damp. Fever is a defense mechanism, and antipyretics should be administered only when the client is uncomfortable. Ice bags can help cool the client quickly but are not comfort measures. Fans are discouraged because they can disperse microbes.

The student nurse caring for clients understands that which factors must be present to transmit infection? (Select all that apply.) a. Colonization b. Host c. Mode of transmission d. Portal of entry e. Reservoir

B, C, D, E Factors that must be present in order to transmit an infection include a host with a portal of entry, a mode of transmission, and a reservoir. Colonization is not one of these factors.

A nurse assesses a newborn 1 minute after birth. The body is pink with blue extremities; the heart rate is 122 beats/min; the legs are withdrawn when the soles are flicked, respiration is easy, with no evidence of distress; and the arms and legs are flexed and moving vigorously. What Apgar score should the nurse document in the newborn's medical record? A.) 7 B.) 8 C.) 9 D.) 10

C --One point was removed from the Apgar score because the extremities are blue. Scores of 7 and 8 are too low and do not reflect the status of the newborn. A score of 10 is too high and does not reflect the status of the newborn.

A nurse is observing as an unlicensed assistive personnel (UAP) performs hygiene and changes a client's bed linens. What action by the UAP requires intervention by the nurse? a. Not using gloves while combing the client's hair b. Rinsing the client's commode pan after use c. Shaking dirty linens and placing them on the floor d. Wearing gloves when providing perianal care

C Shaking dirty linens (or even clean linens) can spread microbes through the air. Placing linens on the floor contaminates the floor surface and can lead to infection spread via shoes. The other actions are appropriate. If the client has a scalp infection or infestation, the UAP should wear gloves; otherwise it is not required.

A nurse is caring for a preterm neonate who is receiving gastric feedings. Which neonatal clinical finding unique to necrotizing enterocolitis (NEC) leads the nurse to suspect that the neonate is experiencing this complication? A.) Persistent diarrhea B.) Decreased abdominal circumference C.) Small amount of vomitus after each gastric feeding D.) Increased amount of residual gastric volume from earlier feedings

D ---An increasing residual volume without increasing intake indicates that absorption is decreasing, a sign of NEC. Diarrhea may or may not be related to NEC. The abdominal circumference increases, not decreases, with NEC. Small amounts of vomitus (spitting up) are common in the neonate because the cardiac (lower esophageal) sphincter of the stomach is weak.

A postpartum nurse is providing care to four maternal/infant couplets. After receiving handoff report from the off-going nurse, which client will the nurse see first? A.) The term infant with a heart rate of 158 beats/min 1 hour after birth B.) The mother who has saturated one peripad over the 4 hours since delivery C.)The mother with a white blood cell count of 12,500/mm3 24 hours after delivery D.) The term infant with a transcutaneous bilirubin reading of 8.6 mg/dL 12 hours after birth

D ---The appearance of jaundice during the first 24 hours of life or persistence beyond the ages delineated usually indicates a potential pathologic process that requires further investigation. The white blood cell count increase is normal after birth, possibly a result of to stress and tissue trauma during the birthing process. The acceptable range for the newborn heart rate is 110 to 160 beats/min. Saturating more than one pad per hour with lochia rubra is a matter of concern because it is less than the acceptable limit.

A preterm neonate admitted to the neonatal intensive care nursery exhibits muscle twitching; seizures; cyanosis; abnormal respirations; and a short, shrill cry. What complication does the nurse suspect? A.) Tetany B.) Spina bifida C.) Hyperkalemia D.) Intercranial hemmorrhage

D --Intracranial bleeding may occur in the subdural, subarachnoid, or intraventricular spaces of the brain, causing pressure on vital centers; clinical signs are related to the area and degree of cerebral involvement. Tetany is caused by hypocalcemia; it is manifested by exaggerated muscle twitching. Spina bifida is a defect of the spinal column that is observed at birth. An increased potassium level causes cardiac irregularities, not the irritable behavior observable with central nervous system involvement.

A nurse is assessing a newborn. Which sign should the nurse report? A.) Temperature of 97.7° F (36.5° C) B.) Pale-pink to rust-colored stain in the diaper C.) Heart rate that decreases to 115 beats/min D.) Breathing pattern with recurrent sternal retractions

D --This infant's breathing pattern is indicative of respiratory distress; the expected pattern is abdominal with synchronous chest movement. A temperature of 97.7° F (36.5° C) is within the expected range of 97.6° F (36.4° C) to 99° F (37.2° C) for a newborn. Pale-pink to rust-colored staining in the diaper is caused by uric acid crystals from the immature kidneys; it is a common occurrence. A decrease in heart rate to 115 beats/min is within the expected range of 110 to 160 beats/min for a newborn.

A hospital unit is participating in a bioterrorism drill. A "client" is admitted with inhalation anthrax. Under what type of precautions does the charge nurse admit the "client"? a. Airborne Precautions b. Contact Precautions c. Droplet Precautions d. Standard Precautions

D Only Standard Precautions are needed. No other special precautions are required for the "client" because inhalation anthrax is not spread person to person.

A client is admitted with possible sepsis. Which action should the nurse perform first? a. Administer antibiotics. b. Give an antipyretic. c. Place the client in isolation. d. Obtain specified cultures.

D Prior to administering antibiotics, the nurse obtains the ordered cultures. Broad-spectrum antibiotics will be administered until the culture and sensitivity results are known. Antipyretics are given if the client is uncomfortable; fever is a defense mechanism. Giving antipyretics does not take priority over obtaining cultures. The client may or may not need isolation.

The nursing instructor explaining infection tells students that which factor is the best and most important barrier to infection? a. Colonization by host bacteria b. Gastrointestinal secretions c. Inflammatory processes d. Skin and mucous membranes

D The skin and mucous membranes are the most important barrier against infection. The other options are also barriers, but are considered secondary to skin and mucous membranes.

Which statements are true regarding Standard Precautions? (Select all that apply.) a. Always wear a gown when performing hygiene on clients. b. Sneeze into your sleeve or into a tissue that you throw away. c. Remain 3 feet away from any client who has an infection. d. Use personal protective equipment as needed for client care. e. Wear gloves when touching client excretions or secretions.

D, E Standard Precautions implies that contact with bodily secretions, excretions, and moist mucous membranes and tissues (excluding perspiration) is potentially infectious. Always wear gloves when coming into contact with such material. Other personal protective equipment is used based on the care being given. For example, if face splashing is expected, you should also wear a mask. Wearing a gown for hygiene is not required. Sneezing into your sleeve or tissue is part of respiratory etiquette. Remaining 3 feet away from clients is also not part of Standard Precautions.

A 28-year-old woman comes into the clinic and tells the nurse that she fears that she is infertile, because she has been trying to become pregnant unsuccessfully for 2 years. While collecting the health history the nurse learns that the client experiences irregular and infrequent menstrual periods. The client is overweight and has severe acne and alopecia. The primary healthcare provider diagnoses the condition polycystic ovarian syndrome (PCOS). Which of the following interventions is the most important? Consoling the client over her inability to have children Discussing weight loss, exercise, and a balanced low-fat diet Providing information to the client on how to prepare for surgery Informing the client that there are no long-term complications of PCOS

Discussing weight loss, exercise, and a balanced low-fat diet Weight loss, exercise, and a balanced low-fat diet can reduce insulin and androgen levels related to PCOS. Meeting with a dietitian may be helpful. Surgery is not necessary at this time. The primary healthcare provider would most likely prescribe hormones, other medications, or both. If pregnancy does not occur, surgery is an option. Pregnancy may be possible with hormones, other medications, or both. Early detection of PCOS is important, because the condition can lead to type 2 diabetes; hypertension; cardiovascular disease; and ovarian, breast, and endometrial cancers. The nurse should encourage treatment compliance and positive lifestyle changes.

A client who has missed two menstrual periods arrives at the prenatal clinic with vaginal bleeding and one-sided lower quadrant pain. Which condition does the nurse suspect? Placenta previa Ectopic pregnancy Incomplete abortion Rupture of a graafian follicle

Ectopic pregnancy A tubal ectopic pregnancy causes first-trimester bleeding; unless an embryo and placenta happen to be located in the abdominal cavity, they cannot grow outside the uterus for longer than 10 to 12 weeks without causing the classic signs of pressure and bleeding. Placenta previa causes painless vaginal bleeding in the third trimester. Abdominal cramping pain is present with an incomplete abortion. A graafian follicle ruptures monthly, without pain or vaginal bleeding, during ovulation; occasionally pain occurs when the follicle ruptures (mittelschmerz), and some vaginal spotting may be noted.

A primigravid client is admitted to the birthing unit in active labor. The fetus is in a breech presentation. Which physiologic response does the nurse expect during this client's labor? Heavy vaginal bleeding Fetal heart rate irregularities Greenish-tinged amniotic fluid Severe back pain with contractions

Greenish-tinged amniotic fluid Greenish amniotic fluid is common in a breech presentation because the contracting uterus exerts pressure on the fetus's lower colon, forcing the expulsion of meconium. Mild bloody show is expected; a heavier flow is a deviation from the expected response and not a common finding with breech presentations. Fetal heart rate irregularities are not specific to a breech presentation. Severe back pain is more likely to occur when the fetus is in a cephalic presentation and the occiput is in the posterior position.

While showing a new mother how to care for her infant's umbilical cord stump, the nurse explains that the stump is a potential source of infection for what reason? Wharton jelly is no longer present. It contains exposed tissue and blood. It is touched by diapers, blankets, and clothing. Newborns do not have immunity to cord infections.

It contains exposed tissue and blood. Exposed tissue and blood in an area that is moist, warm, and dark make an excellent culture medium, so it is important to keep the umbilical area clean and dry. The stump should be cleaned with water, using soap sparingly. Evidence does not support the use of antiseptic preparations for cord care. Wharton jelly is present and provides a protective barrier. The diaper is kept below the level of the umbilicus. Although the site may be touched by clothing, this usually is not a source of bacterial infection. Newborns do have resistance to infections because they carry antibodies from the mother.

A nurse caring for a client who has had a hysterectomy is concerned about the client's risk for postoperative thrombosis. The nurse remembers that the majority of pulmonary emboli begin as deep vein thromboses in what area? Calf Thoracic cavity Pelvis and thighs Extremities and abdomen

Pelvis and thighs Most pulmonary emboli after surgery of the pelvic floor originate in the deep veins of the pelvis and thighs because of the extensive vascular network in the region. The calf, thoracic cavity, extremities, and abdomen are not where most pulmonary emboli originate after surgery involving the pelvic floor.

The nurse is reevaluating a newborn who had an axillary temperature of 97° F (36.1° C) and was placed skin to skin with the mother. The newborn's axillary temperature is still 97° F (36.1° C) after 1 hour of skin-to-skin contact. Which intervention should the nurse implement next? Placing the newborn under a radiant warmer in the nursery Checking the newborn for a wet diaper and then continue the skin-to-skin contact Leaving the newborn in skin-to-skin contact and rechecking the temperature in 1 hour Double-wrapping the newborn in warm blankets and returning the newborn to a crib by the mother's bedside

Placing the newborn under a radiant warmer in the nursery The newborn's temperature should be kept in the normal range of 97.7° F to 99.5° F (36.5° C to 37.5° C). A hypothermic temperature that has not improved in 1 hour with the use of skin-to-skin contact requires additional measures. The infant should be placed under a radiant warmer for a short time until the temperature returns to the normal range. Continuing skin-to-skin contact would not resolve the problem of hypothermia. Double-wrapping the newborn in warm blankets and leaving the newborn at the bedside would not be an adequate means of resolving the hypothermia.

While auscultating the lungs of a client admitted with severe preeclampsia, the nurse identifies crackles. What inference does the nurse make when considering the presence of crackles in the lungs? Seizure activity is imminent. Pulmonary edema has developed. Bronchial constriction was precipitated by the stress of pregnancy. Impaired diaphragmatic function was caused by the enlarged uterus.

Pulmonary edema has developed. Pulmonary edema is associated with severe preeclampsia; as vasospasms worsen, capillary endothelial damage results in capillary leakage into the alveoli. Crackles are not an indication of an impending seizure; signs of an impending seizure include hyperreflexia, developing or worsening clonus, severe headache, visual disturbances, and epigastric pain. Pregnancy does not precipitate bronchial constriction, although the hormones associated with pregnancy can cause nasal congestion. Impaired diaphragmatic function is a discomfort associated with pregnancy that may result in shortness of breath or dyspnea, not crackles.

A nurse is obtaining a health history from a client with newly diagnosed cervical cancer. Which aspect of the client's life is most important for the nurse to explore at this time? Sexual history Support system Obstetric history Elimination patterns

Support system During a health crisis the client will need support from significant others. The sexual history is important in diagnosis and the obstetric history and elimination patterns are important parts of the medical history; however, none are the priority at this time.

The nurse is caring for a preterm infant who is receiving oxygen therapy. What should the nurse do to prevent retinopathy of prematurity (ROP)? Cover the neonate's eyes with a shield Place the neonate in an elevated side-lying position Assess the neonate every hour with a pulse oximeter Support the neonate's oxygen saturation while providing minimal FiO2

Support the neonate's oxygen saturation while providing minimal FiO2 ROP is a complex disease of the preterm infant; hyperoxemia is one of the numerous causes implicated. Oxygen therapy is maintained at the lowest level necessary to support respiratory status. If the oxygen concentration needs to be increased to maintain life, ROP may not be preventable. Using a shield over the neonate's eyes will not prevent the development of ROP; nor does positioning or assessment of the neonate every hour with a pulse oximeter alone. If the pulse oximetry results are within an acceptable range, the oxygen concentration may be reduced.

A client is admitted with fever, myalgia, and a papular rash on the face, palms, and soles of the feet. What action should the nurse take first? a. Obtain cultures of the lesions. b. Place the client on Airborne Precautions. c. Prepare to administer antibiotics. d. Provide comfort measures for the rash.

This client has manifestations of smallpox, a public health emergency, and should be placed on Airborne Precautions first before other care measures are implemented.


संबंधित स्टडी सेट्स

CCNA Chapter 4 Practice Questions

View Set

A Sociology of the Family Inquisitive

View Set

Business Finance Ch 11 Homework - Connect

View Set

Chapter 9 - The Post Effective Period

View Set

Глава 1: Знакомство с колледжем

View Set

Lecture Exam 3: Reproductive (updated 4.10.15)

View Set

anatomy common and scientific name (head, neck, and back)

View Set

Chapter 11 - Shareholders' Equity (Financial Accounting)

View Set

Anatomy ch 13Functions of the nervous system include which of the following?

View Set

HESI Questions Part 1 - Fundamental Skills

View Set

Computer History, Computer Knowledge

View Set

Quiz: CompTIA Network+ N10-008 Post-Assessment Quiz

View Set

Экспериментальна психологія

View Set

how to highlight when annotating

View Set